You are on page 1of 67

DENTAL

QUEST

KCET -2011 66

K-CET-2011
HUMAN ANATOMY AND EMBRYOLOGY
1)
Which of the following muscle is attached to posterior part of Pterygomandibular raphe? a) Lateral pterygoid b) Buccinator c) Superior constrictor d) Medial pterygoid Ans (c) Superior Constrictor (Grays anatomy 39th ed pg no. 629) It gives attachment posteriorly to the superior constrictor of the pharynx, and anteriorly to the central part of buccinator.

K C E T 2 0 1 1

2)

What is the type of joint formed between the tooth and the alveolar socket ? a) Gomphosis b) Syndesmosis c) Suture d) Synovial Ans (a) Gomphosis (B.D Chaurasias Handbook of General anatomy 3rd edition page no 50) Structural Classification of joints: o o o Fibrous joints Sutures, Syndesmosis and Gomphosis (e.g tooth in its socket) Cartilaginous joints Primary (synchondrosis) Secondary (symphysis) Synovial joints

3) The classic karyotype of Klinefelter syndrome is a) 45, XO b) 47, XXY c) Trisomy 18 d) Trisomy 21 Ans (b) 47, XXY (Human Embryology by Inderbir Singh 7th ed page no 24) Presence or absence of an extra X or Y chromosome can give rise to various syndromes associated with abnormal genital developmental retardation and abnormal growth. Some of these are XXX(abnormal female),XXY (klinefelters syndrome;abnormal male) and XYY(abnormal male) In Klinefelters syndrome, the subject is a male (because of the presence of a Y chromosome),though testes are poorly developed leading to sterility and Gynecomastia. 4) Philtrum of upper lip develops from a) Mandibular process b) Maxillary process c) Frontonasal process d) Palatal process Ans (c) Frontonasal Process (Human embryology by inderbir singh 7th ed page no 139) Lower lip: - Along with lower jaw it is formed by the fusion of mandibular processes in the midline that grows towards each other.

DENTAL QUEST Your Quest For The success In Post Graduate Dental Entrance !

DENTAL

QUEST

KCET -2011 67

Upper lip: - Each maxillary process grows medially and fuses first with the lateral nasal process and then with the median nasal process. The mesodermal basis of lateral part of lip is formed from the maxillary process where as the mesodermal basis of median part of lip (philtrum) is formed from the frontonasal process.

PHYSIOLOGY
5) The pH of gastric juice is about a) 1 b) 6 c) 9 d) 7 Ans (a) 1 (Ganong's Review of Medical Physiology, 22ed page no 491) The cells of gastric glands secrete about 2500 ml of gastric juice daily. The HCl secreted by glands in the body of stomach provides the necessary pH for pepsin to start protein digestion and stimulates bile flow. The pH gradient established have a range of pH 1.0-2.0 at the luminal side as compared to pH of 6.0-7.0 at the surface of epithelial cells.

K C E T 2 0 1 1

6)

Ceruloplasmin is a protein containing? a) Iron b) Copper c) Manganese d) Magnesium Ans (b) Copper (Ganong's Review of Medical Physiology, 22ed page no 214,541) Also remember Wilson disease or hepatolenticular degeneration A genetic autosomal recessive disorder of copper metabolism in which plasma level of copperbinding protein ceruloplasmin is usually low and there is chronic copper intoxication and severe degeneration of lenticular nucleus. Mutation on the long arm of chromosome 13q. Affects the copper-transporting ATPase gene (ATP7B) in the liver, leading to an accumulation of copper in the liver and resultant progressive liver damage. Copper accumulates in the periphery of the cornea in the eye accounting for the characteristic yellow KayserFleischer rings. Motor disturbances include "wing-beating" tremor or asterixis, dysarthria, unsteady gait, and rigidity. 7) Parathyroid hormone a) Decreases the intestinal calcium absorption b) Stimulates bone resorption c) Stimulates osteoblasts d) Increases renal excretion of calcium Ans (b) Stimulates bone resorption (Ganong's Review of Medical Physiology, 22ed page no 394) The normal plasma level of PTH is 10 to 55 pg/Ml and the half-life of PTH is approximately 10 min. Actions PTH acts directly on bone to increase bone resorption and mobilize Ca2+.

DENTAL QUEST Your Quest For The success In Post Graduate Dental Entrance !

DENTAL

QUEST

KCET -2011 68

In addition to increasing the plasma Ca 2+, PTH increases phosphate excretion in the urine and thereby depresses plasma phosphate levels PTH also increases the formation of 1, 25-dihydroxycholecalciferol, and this increases Ca2+ absorption from the intestine. On a longer time scale, PTH stimulates both osteoblasts and osteoclasts. The net effect of mildly elevated plasma PTH levels it is usually anabolic. 8) The acrosome reaction occurs when the sperm a) Enters the uterine cavity b) Comes in contact with zona pellucida c) Penetrates corona radiata d) Penetrates cell membrane of oocyte Ans (b) comes in contact with zona pellucida (Ganong's Review of Medical Physiology, 22ed page no 449)

K C E T 2 0 1 1

Sperms bind to a sperm receptor in the zona, and this is followed by the acrosomal reaction, that is, the breakdown of the acrosome, the lysosome-like organelle on the head of the sperm 9) The first heart sound is due to a) Sudden closure of mitral and tricuspid valve b) Sudden closure of semilunar valves c) Ventricular systole d) Atrial systole Ans (a) sudden closure of mitral and bicuspid valve (Ganong's Review of Medical Physiology, 22ed page no 569) Heart Sounds Two sounds are normally heard through a stethoscope during each cardiac cycle. The first is a low, slightly prolonged "lub" , caused by sudden closure of the AV valves at the start of ventricular systole. Duration of about 0.15 s and a frequency of 25 to 45 Hz. The second is a shorter, high-pitched "dup" (second sound), caused closure of the aortic and pulmonary valves just after the end of ventricular systole. Duration of about 0.12 s, with a frequency of 50 Hz. A soft, low-pitched third sound is heard coinciding with the period of rapid ventricular filling and is due to vibrations set up by the inrush of blood. It has duration of 0.1 s. A fourth sound can sometimes be heard immediately before the first sound when atrial pressure is high or the ventricle is stiff in conditions such as ventricular hypertrophy. It is due to ventricular filling and is rarely heard in normal adults.

10)

Which one of the following is a vasodilator? a) Bradykinin b) Norepincphrin c) Epinephrine d) Angiotensin II Ans (a) Bradykinin (Ganong's Review of Medical Physiology, 22ed page no 601) Many circulating hormones affect the vascular system. The vasodilator hormones include Kinins,VIP, and ANP. Circulating vasoconstrictor hormones include vasopressin, norepinephrine, epinephrine, and angiotensin II.

11)
a) b) c) d)

The condition, in which the person can see near objects but not the far objects, is? Hypermetropia Myopia Presbyopia Astigmatism

DENTAL QUEST Your Quest For The success In Post Graduate Dental Entrance !

DENTAL

QUEST

KCET -2011 69

Ans (b) Myopia (Ganong's Review of Medical Physiology, 22ed page no 157)

Myopia (nearsightedness- Can see the near objects)- the antero-posterior diameter of the eyeball is too long, corrected by glasses with biconcave lenses. Astigmatism - curvature of the cornea is not uniform. Corrected with cylindrical lenses. Presbyopia - Progressively diminished ability to focus on near objects .It can be corrected with glasses or contact lenses Hyperopia or hypermetropia (farsightedness), the eyeball is too short and light rays come to a focus behind the retina. A biconvex lens corrects this by adding to the refractive power of the lens of the eye. 12) The male sex hormone testosterone is produced by a) Sertoli cells b) Epithelial cells c) Interstitial cells of leydig d) Primitive germ cells Ans (c) Interstitial cells of leydig (Ganong's Review of Medical Physiology, 22ed page no 428) Testosterone:Chemistry & Biosynthesis Testosterone, the principal hormone of the testes, is a C19 steroid with an OH group in the 17 position. It is synthesized from cholesterol in the Leydig Cells and is also formed from androstenedione secreted by the adrenal cortex. The secretion of testosterone is under the control of LH, and the mechanism by which LH stimulates the Leydig cells involves increased formation of C-AMP via the G protein-coupled LH receptor and Gs. Secretion The testosterone secretion rate is 4 to 9 mg/d (13.931.33 mol/d). Transport & Metabolism Ninety-eight percent of the testosterone in plasma is bound to protein: 65% is bound to alpha -globulin called gonadal steroid-binding globulin (GBG) or sex steroid-binding globulin, and 33% to albumin. Actions In addition to their actions during development, testosterone and other androgens exert An inhibitory feedback effect on pituitary LH secretion Develop and maintain the male secondary sex characteristics Exert an important protein-anabolic, Growth-promoting effect and Along with FSH, maintain spermatogenesis.

K C E T 2 0 1 1

BIOCHEMISTRY
13)
In prolonged starvation the main source of energy to brain is? a) Glucose b) Fructose c) Fatty acids d) Ketone bodies Ans (d) ketone bodies (Harpers illustrated biochemistry 25th ed page no 124) Acetyl-CoA formed by -oxidation may undergo several fates:

DENTAL QUEST Your Quest For The success In Post Graduate Dental Entrance !

DENTAL

QUEST

KCET -2011 70

As with acetyl-CoA arising from glycolysis, it is oxidized to CO2 + H2O via the citric acid cycle. It is the precursor for synthesis of cholesterol and other steroids. In the liver, it forms ketone bodies (acetone, acetoacetate, and 3-hydroxybutyrate) that are important fuels in prolonged starvation. 14) LDH has following number of isoenzymes a) 5 b) 3 c) 11 d) 2 Ans (a) 5 (Harper's biochemistry 25th ed page no 57) L-Lactate dehydrogenase is a tetrameric enzyme whose four subunits occur in two isoforms, designated H (for heart) and M (for muscle). The subunits can combine to yield catalytically 5 active isozymes of L-lactate dehydrogenase: Lactate Dehydrogenase L1 L2 L3 L4 L5 Isozyme Subunits HHHH HHHM HHMM HMMM MMMM

K C E T 2 0 1 1

Distinct genes whose expression is differentially regulated in various tissues encode the H and M subunits. Heart expresses the H subunit almost exclusively, isozyme I1 predominates in this tissue. Isozyme I5 predominates in liver. Following a myocardial infarction or in liver disease, the damaged tissues release characteristic lactate dehydrogenase isoforms into the blood. 15) The quantity of water lost in expired air in each 24 hrs is about? a) 200ml b) 400ml c) 600 ml d) 800 ml Ans (b) 400ml (Biochemistry by U.Satyanarayana page no.520) Distribution of water Average 60% of body weight , Men 55-70%, Women 45-60 % Average 70 kg normal man contains 24 lit of water Water intake Exogenous water- 0.5-5 lit controlled by thirst center and hypothalamus Endogenous water- 300-350ml/day Water output Urine 1-2l/day Lungs- 400ml/day Skin- 450ml/day Feces- 150ml/day The loss of water by via skin and lungs is called as insensible water loss. DENTAL MATERIALS a) 16) Which form of stainless steel is used for the manufacturing of orthodontic wire Austenitic

DENTAL QUEST Your Quest For The success In Post Graduate Dental Entrance !

DENTAL

QUEST

KCET -2011 71

b) Martenstic c) Ferritic d) All the three form can be used Ans (a) Austenite (Phillips science of dental materials 11th ed page no 639) Austenitic steel is used for dental application because of the following excellent properties: Greater ductility and ability to undergo , more cold workable without fracturing. Substantial strengthening during cold working (some transformation to martensitic phase) Greater ease of welding. Ability to overcome sensitization. Less critical grain growth. Ease of formation. 17) Which one of the following restorative materials is having coefficient of thermal expansion close to that of tooth enamel and dentin ? a) Microfilled composite resins b) Pure gold c) Silver amalgam d) Type II Glass lonomer cement Ans (d)Type II Glass Ionomer cement (Phillips science of dental materials 11th ed page no 55 and Craigs Restorative Dental Materials, 11th edition pg no 53) LINEAR COEFFICIENT OF THERMAL EXPANSION OF VARIOUS MATERIALS. Material Inlay waxes Silicone impression material Polysullide impression material Pit and fissure sealants Acrylic resin Mercury Resin composites Zinc oxide-eugenol cement Amalgam Silver Copper Gold Porcelain Tooth (crown portion) Glass ionomer (type 2) coefficient x 10-6/C 350-450 210 140 71-94 76.0 60.6 14-50 35 22-28 19.2 16.8 14.4 12.0 11.4 10.2-11.4

K C E T 2 0 1 1

18)

Which of the following impression materials is easy to pour and difficult to remove the stone cast from the impressions? a) Addition polysilicone b) Condensation polysilicone c) Polyether d) Poly sulfide Ans (c) Polyether (Phillips Science of dental materials 11th Ed page no 227) The stiffness increases as following order:-polysulfides,condensation silicones,addition silicone and polyether.

DENTAL QUEST Your Quest For The success In Post Graduate Dental Entrance !

DENTAL

QUEST

KCET -2011 72

The polyether is difficult to remove from the undercut areas because of very high modulus of elasticity. 19) As per A. D. A No. 25, minimum amount of setting expansion required for type V gypsum product is a) 0.05 b) 0.10 c) 0.15 d) 0.20 Ans (b) 0.10 (Phillips Science of dental materials 11th ed page no 273, Craig restorative dental materials,11th edition pg no 397)

K C E T 2 0 1 1

20)

Polyacid modified composites are designated as? a) Compomers b) Giomers c) Hybrid ionomers d) Ormocers Ans (a)Compomers (Phillips science of dental materials 11th ed page no 472, Craigs restorative dental materials 11 th edition pg no 244) They are the hybrids of dental composites and glass ionomer cement. They are also known as polyacid-modified resin composites. COMPOMERS Compomers or poly acid-modified composites are used for restorations in low stress-bearing areas. Compomers are recommended for patients at medium risk of developing caries. COMPOSITION AND SETTING REACTION Compomers contain poly acid-modified monomers with fluoride-releasing silicate glasses and are formulated without water. The volume percent filler ranges from 42% to 67% and the average filler particle size ranges from 0.8 to 5.0 micrometer. Setting occurs primarily by light-cured polymerization, but an acid-base reaction also occurs as the compomer absorbs water after placement and upon contact with saliva. PROPERTIES Compomers release fluoride by a mechanism similar to that of glass and hybrid ionomers. Because of the lower amount of glass ionomer present in compomers, the amount of fluoride release and its duration are lower than those of glass and hybrid ionomers. Also, compomers do not recharge from fluoride treatments or brushing with fluoride dentifrices as much as glass and hybrid ionomers.

21)

Solder and flux used for joining Elgiloy wires are? a) Gold solder and borax flux b) Gold solder and fluoride flux c) Silver solder and borax flux d) Silver solder and fluoride flux Ans (d)Silver solder and fluoride flux (Phillips science of dental materials 11th ed page no 609 611 612) Use of Gold solder with Elgiloy- a type of Nickel titanium derivative will form an layer of an alloy which will melt at 9000 . Hence Use of Silver Solder is indicated. The oxide layer formed on this highly reactive metal is more stable and borax flux is not able to dissolve this layer as they are just protective flux.

22)

Ceramics fired to metals are processed by?

DENTAL QUEST Your Quest For The success In Post Graduate Dental Entrance !

DENTAL

QUEST

KCET -2011 73

a) Heat pressing b) Machining c) Sintering d) Slip casting Ans (c)Sintering (Craigs restorative dental materials 11th ed, pg no 553-554,Phillips science of dental materials 11th ed page no 665)

One of the most common fabrication techniques for dental ceramics is called sintering. Sintering is the process of heating the ceramic to ensure densification. This occurs by viscous flow when the firing temperature is reached. Ceramics fired to metals are processed by sintering, whereas all-ceramic materials encompass a wider range of processing techniques, including heat-pressing, machining, and slipcasting.

K C E T 2 0 1 1

23)

A true eutectic alloy has melting point? a) Above that of low fusing metal b) Above the melting point of either metal c) Below that of high fusion metal d) Below the melting point of either metal Ans (d) Below the melting point of either metal (Craigs restorative dental materials 11th ed page no 172,) A eutectic alloy system occurs when two metals are soluble as liquids but nearly insoluble as solids. These systems have a single composition (the eutectic composition) with a melting point that is lower than either component metal.

24)

All of the following components are present in the liquid composition of the heat cure acrylic resins EXCEPT? a) Glycol dimethacrylate b) Hydroquinone c) Methyl methacrylate d) N, N - dihydroxy ethyl para toluidine Ans (d) N,N-dihydroxy ethyl para toluidine (Phillips science of dental materials 11th ed page no 723) Most (poly) methyl methacrylate resin systems include powder and liquid components. Powder: Prepolymerized spheres of polymethylmethacrylate Benzoyl (initiator). peroxide Liquid Non polymerized methacrylate Hydroquinone (inhibitor). Glycol dimethacrylate (cross linking agent) methyl

25)

Direct filling gold formed by electrolytic precipitation is called by all the terms EXCEPT ? a) Crystalline gold b) Granular gold c) Mat gold d) Sponge gold Ans (b) Granular gold

DENTAL QUEST Your Quest For The success In Post Graduate Dental Entrance !

DENTAL

QUEST
th

KCET -2011 74

(Phillips science of dental materials 11 ed page no 547) DFGs that are currently available are divided into 3 categories 1. Foil (also known as fibrous gold) 2. Electrolytic precipitate (also called crystalline gold, sponge gold or mat gold ) 3. Granular gold (also called powdered gold). 26) The dominant phase in a well - condensed, low copper dental amalgam is Ag2Hg3 Ag3Sn c) Cu,Sn d) Sn8Hg Ans (a)Ag2Hg3 (Operative dentistry Modern theory and Practice by Marzouk page no.106)

a) b)

K C E T 2 0 1 1

o o

PHASES OF AMALGAM Ag3Sn (gamma ) Mechanically it is the strongest

Ag2Hg3 (gamma 1) Nobelest phase,most resistant to tarnish and corrosion.every effort should be made to keep this in maximum available space in the bonding matrix of final products. Sn8Hg(Gamma 2) o Least resistant to tarnish and corrosion. o Responsible for failure of restoration Hg phase( Mercury phase)- remember this point o mechanically the weakest phase Other phases: i) Void(pore) ii) Trace element phase iii) Interphase 27) Which glass ionomer cement develops early resistance to water intake on setting? a) Reinforced auto cure glass ionomer cement b) Compomer c) Resin modified glass ionomer cement d) Ceramic reinforced glass ionomer cement Ans (c)Resin modified glass ionomer cement ( Phillips science of dental materials 11th ed page no 482) Resin modified GIC( Hybrid Inomer) Powder contains o Ion leachable fluoroaluminosilicate glass particle and initiator for curing Liquid contains polyacid with methacrylate and hydroxyethyl methacylate Setting reactiono Initial setting by polymerization o Final set of conventional acid base setting Advantages o Less moisture sensitivity o Greater early strength o Improved translucency o Higher dimetrile strength o Lower elastic modulus o Greater plastic deformation o Greater bond strength when used with composite Disadvantage o More Polymerization shrinkage and micro leakage o Lesser wetting capacity

DENTAL QUEST Your Quest For The success In Post Graduate Dental Entrance !

DENTAL
o

QUEST

KCET -2011 75

Transient temperature increase while setting

PHARMACOLOGY
28)
An antibiotic that achieves high concentration in Hard tissue, because of its Molecular size is? a) Ceftum b) Ciprofloxacin c) Clindamycin d) Erythromycin Ans (c) Clindamycin (Oral and maxillofacial infections By Richard G. Topazian 4th ed page no 120) A patient presents with congestive cardiac failure with depressed ejection fraction. Which drug may be effective in reducing mortality? a) Digoxin b) Frusemide c) Lidocaine d) Enalapril Ans (d) Enalpril (K.D. Tripathi Essentials of Medical Pharmacology 5th ed page no 468) Enalapril is an ACE inhibitor used for long term CHF cases. They have been found to be effective in reducing mortality in these patients.

K C E T 2 0 1 1

29)

30)
a) b) c) d)

The most common adverse effect associated with phenytoin is? Megaloblastic anemia Gum hypertrophy Hirsutism Drowsiness

Ans (b) Gum hypertrophy (Essentials of pharmacology K.D.Tripathi 6th ed pg 404 ) Adverse effects of Phenytoin o Gum hypertrophy commonest (20%) o Hirsutism o Acne o Hypersensitivity reaction

Megaloblastic anemia o Osteomalacia o Foetal hydantoin syndrome

31)
a) b) c) d)

Morphine withdrawal is characterized by all EXCEPT Miosis Yawning Lacrimation Diarrhea

Ans (a)Miosis (ESSENTIALS OF PHARMACOLOGY K.D. Tripathy 6th ed pg 457) Withdrawal symptoms of morphine Lacrimation Sweating Yawning Anxiety Gooseflesh Mydriasis Dehydration Rise in B.P.

DENTAL QUEST Your Quest For The success In Post Graduate Dental Entrance !

DENTAL

QUEST
Absence convulsion

KCET -2011 76

Palpitation Rapid weight loss

of

delirium

and

32) Ethambutol causes a) Red discoloration of urine b) Psychosis c) Optic neuritis d) Flu like syndrome Ans (c)Optic neuritis (K.D. Tripathi Essentials of Medical Pharmacology 5th ed page no 421) SIDE EFFECTS OF ETHAMBUTOL Loss of visual acuity or colour vision due to optic neuritis and is dose Also causes Hyperuricemia 33) Ipratropium bromide is used for a)Renal colic b)Organophosphorus poisoning c)Bronchial asthma d)Miosis Ans (c)Bronchial asthma (K.D. Tripathi Essentials of Medical Pharmacology 5th ed page no 97) Ipratropium bromide is an anticholinergic drug which acts selectively on bronchial muscles but does not affect respiratory secretion. In contrast to sympathomimetic which acts on peripheral bronchioles, ipratropium bromide mainly acts on larger central airways. Although it is used for treatment of bronchial asthma but is more effective in COPD. 34) Drug used in treatment of malignant hyperthermia is a) Baclofen b) Diazepam c) Dantrolene d) Halothane Ans (c)Dantrolene (K.D. Tripathi Essentials of Medical Pharamcology 5th ed page no 316) Dantrolene is a directly acting muscle relaxant. It is given I.V. for treating malignant hyperthermia. 35) Which of these drugs can be given safely to a patient with renal disease ? a) Phenacetin b) Tetracycline c) Aminoglycoside d) Diazepam Ans (d)Diazepam (Essentials of pharmacology by K.D.Tripathi 5th ed pg no 365) Phenacitin causes analgesic nephropathy. Tetracycline causes kidney damage except doxycycline which is named as Phanconis syndrome Aminoglycosides results in nephrotoxicity due to tubular damage. Diazepam is a BZD which is relatively a safe drug. dependent.

K C E T 2 0 1 1

DENTAL ANATOMY

DENTAL QUEST Your Quest For The success In Post Graduate Dental Entrance !

DENTAL 36)

QUEST

KCET -2011 77

"Y" shaped arrangement of central groove is seen on the Occlusal surface of a) Permanent maxillary first premolar b) Permanent maxillary second premolar c) Permanent mandibular first premolar d) Permanent mandibular second premolar Ans (d) Permanent mandibular second premolar (Wheelers dental anatomy physiology and occlusion 6th ed page no 213)\

There are two common Occlusal forms of mandibular 2nd premolar in which most often seen is three cusp type than less common two cusp type. The two types differ mainly from Occlusal aspect. In the three cusp type the buccal cusp is the largest followed by mesiolingual and distolingual cusp. Each cusp has well-formed triangular ridges separated by deep developmental grooves These grooves converges in a central pit and forms Y- on the Occlusal surface 37) The cusp of carebelli of the maxillary first molar is on a) Mesiobuccal cusp b) Mesiolingual cusp c) Distobuccal cusp d) Distolingual cusp Ans (b)Mesiolingual cusp (Wheelers dental anatomy physiology and occlusion 6th ed page no 65, 217) It is a supplemental cusp which varies in size from small cusp to a small indented pit or fissure Most common location in permanent dentition is Mesio - palatal cusp of maxillary permanent 1st molar and in primary is deciduous 2nd molars. 38) Which of the following teeth have a mesio-distal dimension greater than the cervico incisal dimension ? a) Permanent maxillary central incisor b) Primary lower central incisor c) Primary lower lateral incisor d) Primary maxillary central incisor Ans (d)Primary maxillary central incisor (Wheeler's Dental Anatomy, Physiology, and Occlusion. 8th ed page no 79) Major Differences Between Primary and Permanent teeth The crown of primary anterior are wider mesiodistally in comparison with the crown length than are the permanent anterior tooth. Roots are narrower and longer and slender with a more flaring than the permanent teeth Primary teeth have prominent cervical ridge Crown are more slender at cervical portion of crown in primary teeth Primary teeth have flatter buccal and facial walls and have small Occlusal table Primary teeth are more whiter in appearance. DENTAL HISTOLOGY

K C E T 2 0 1 1

39)

"Von Ebner" salivary glands are serous glands present in association with? a) Filliform Papilla b) Circumvallate Papilla c) Foliate Papilla d) Fungiform Papilla Ans (b)Circumvallate papilla

DENTAL QUEST Your Quest For The success In Post Graduate Dental Entrance !

DENTAL

QUEST
th

KCET -2011 78

( Orbans oral histology and embryology 11 ed page no. 360) Glands of tongue are divided into various groups. Anterior regions are mucous in nature while posterior are mixed. Anterior lingual glands(glands of Blandin and Nuhn) are located near apex of tongue. Posterior lingual mucous glands are located lateral and posterior to vallate papillae and opens onto dorsal surface of tongue The posterior lingual serous glands(von ebners) are located below vallate papillae and their ducts opens into the trough of vallate papillae and at the rudimentary foliate papillae on the sides of tongue 40) Dark bands that are present in enamel rods at intervals of 4 mms are a) Neonatal bands b) Cross striations c) Striae of Retzius d) Hunter - Schregar bands Ans (b)Cross striations ( Orbans oral histology and embryology 11th ed page no. 57)

K C E T 2 0 1 1

Each enamel rod is made up of segments separated by dark lines that give it a striated appearance. Becomes more visible on application of mild acids More pronounced in insufficiently calcified enamel Rods are segmented because matrix is formed in a rhythmic manner

41)

In reflected light, dead tracts are seen as? a) White zones b) Black zones c) Purple zones d) A combination of white & black zones Ans (b)Black zones (Orban's Oral Histology and Embryology 11th ed page no. 125) The normal odontoblastic processes disintegrate due to caries, attrition, erosion or during formation of ground section which is filled with air. These degenerated odontoblastic processes give raise to dead tracks which appear black in transmitted light and white in reflected light. More commonly seen in pulpal horn areas due to crowding of odontoblasts in these area. Whereas sclerotic dentin appears light in transmitted light and black in reflected light. 42) Merkel cells are found in which of the following tissue ? a) Epidermis b) Papillary layer of dermis c) Reticular layer of dermis d) Hypodermis Ans (a)Epidermis (Orbans oral histology and embryology 11th Ed page no. 297) Oral epithelium contains three types of cells apart from normal keratinocytes. These are a) Melanocytes (derived from neural crest) b) Langerhans cells (originated in bone marrow) and c) Merkel cells. It is believed to be a specialized neural pressure-sensitive receptor cells believed to be of neural crest origin. GENERAL PATHOLOGY

DENTAL QUEST Your Quest For The success In Post Graduate Dental Entrance !

DENTAL

QUEST

KCET -2011 79

43) Somatic mutation of PTEN is seen in a) Retinoblastoma b) Osteosarcoma c) Carcinoma breast d) Endometrial carcinoma Ans (d)Endometrial carcinoma (Robbins pathology 7th edition pg no.693)

Mutations in the PTEN tumor suppressor gene are found in 30% to 80% of endometrioid carcinomas and in 20% of endometrial hyperplasias. PTEN mutations occur before the development of invasion.

K C E T 2 0 1 1

OTHER IMPORTANT TUMOUR SUPPRESSOR GENES INVOLVED IN HUMAN NEOPLASIA RB1Retinoblastomas, osteosarcoma P16/INK4a.Malignant melanoma BRCA1 and BRCA2Carcinomas of female breast and ovary; carcinomas of male breast E-cadherinFamilial gastric cancer 44) Tigered effect of heart is seen in a) Fatty change b) Hyaline change c) Amyloidosis d) Atrophy Ans (a)Fatty change (Robbins & Cotran Pathologic Basis of Disease, 7th ed page no 36)

Steatosis and fatty change is the abnormal accumulations of triglycerides within parenchymal

cells. Fatty change is often seen in the liver because it is the major organ which is involved in fat metabolism. It also occurs in heart, muscle, and kidney. The causes of steatosis include toxins, protein malnutrition, diabetes mellitus, obesity, and anoxia. In prolonged moderate hypoxia, such as that produced by profound anemia, causes intracellular deposits of fat, which create grossly apparent bands of yellowed myocardium alternating with bands of darker, red-brown, uninvolved myocardium (tigered effect). 45) a) b) c) d) Carcinoembryonic antigen is associated with Hepatocellular carcinoma Medullary carcinoma of thyroid Seminoma Carcinoma of colon

Ans (d) Carcinoma of colon (ROBBINS PATHOLOGY 7th edition pg nu 208,585) Selected tumour markers CEA- Carcinoma of colon, pancreas, lung, Stomach and heart Calcitonin- Medullary carcinoma of thyroid Alpha feto protein - Liver cancer 46) a) b) Which of the following is NOT a constituent of Virchow's triad ? Damage to endothelium due to injury or inflammation Diminished rate of blood flow

DENTAL QUEST Your Quest For The success In Post Graduate Dental Entrance !

DENTAL

QUEST

KCET -2011 80

c) Increased coagulabilty of blood d) Increased venous blood pressure Ans (d) Increased venous blood pressure (Robbins & Cotran Pathologic Basis of Disease, 7th ed page no 90) Thrombosis is the inactivation of blood clotting in uninjured vasculature or thrombotic occlusion of vessel after relatively minor injury Three factor primarily influence the formation of thrombosis is termed as Virchows Triad. They are-

Endothelial injury, a major factor mainly in heart and arterial circulation . several causes like hypertension, or smoking, high cholesterol level may lead to injury which leads to series of events like adherence of platelets to exposed subendothelial matrix Alteration in normal blood flow mainly seen as disturbance in laminar blood flow pattern resulting is stasis of blood hence causing thrombosis. Hypercoagulability , a minor factor, most commonly due to factor V gene mutation. Types of thrombi Thrombi in cardiovascular system- they are most of the time retrograde extension from point of attachment.

K C E T 2 0 1 1

Nonocclusive(Mural)- aortic or cardiac thrombi ,Gray red, have line of Zahn produced by layers of platelet and fibrin with darker red cell rich layer. Occlusive thrombi small arterial thrombi

-Venous thrombi- (Phlebothrombosis)- they are most of the time extend towards the direction of blood flow from point of attachment and have more number of RBC -Thrombi can also occur on heart valves- In infective endocarditis, of fungal infection, disseminated cancer and verrocous endocarditis. MICROBIOLOGY 47) In which phase of the Bacterial growth curve involution forms of the bacteria can be observed ? a) Lag phase b) Log phase c) Stationary phase d) Phase of decline Ans (d) Phase of decline (Text Book of Microbiology by Ananthanarayan 7th ed page no 21) Bacterial growth can be modeled with four different phases: lag phase (A), exponential or log phase (B), stationary phase (C), and death or decline phase (D).

1.

2.

Lag phase: No appreciable increase in number There may be an increase in size of the cell. Bacteria has maximum cell size towards the end of this phase. Exponential or Log(logarithmic phase): Characterized by cell doubling. The number of new bacteria appearing per unit time is proportional to the present population. Plotting the natural logarithm of cell number against time produces a straight line.

DENTAL QUEST Your Quest For The success In Post Graduate Dental Entrance !

DENTAL

QUEST
Cells are smaller and stains uniformly.

KCET -2011 81

3.

Stationary phase : The growth rate slows as a result of nutrient depletion and accumulation of toxic products. Value is constant as the rate of bacterial growth is equal to the rate of bacterial death. Cells show irregular gram staining due to presence of intracellular storage granules. Sporulation occurs in this stage.

K C E T 2 0 1 1

4. Death phase or phase of decline : Bacteria run out of nutrients and die.
Involution occurs in this phase. 48) Out of the following hypersensitivity reactions, in which type a single dose of the antigen can act as both the sensitizing and shocking dose. a) Anaphylaxis b) Arthus reaction c) Serum sickness d) Contact dermatitis Ans (c)Serum sickness (Text Book Of Microbiology by Ananthanarayan 7th ed page no 165) Serum sickness is a systemic form of type 3 hypersensitivity. Described by Von Pirquet and Schick Appears 7-12 days after a single injection of high concentration of foreign serum Pathogenesis is the formation of immune complexes that gets deposited on endothelial lining of blood vessels in various parts of the body. Serum sickness differs from other type of hypersensitivity reaction in that a single dose of the antigen can serve as both the sensitizing and shocking dose. Seen more commonly following penicillin or other antibiotics injections 49) Zichl Neelsen's staining is routinely performed from sputum samples for the detection of a) Mycobacterium tuberculosis b) Bacillus anthracis c) Streptococcus pneumoniae d) Mycoplasma pneumonia Ans (a) Mycobacterium tuberculosis (Text Book Of Microbiology by Ananthanarayan 7th ed page no 351) The ZiehlNeelsen stain is also known as the acid-fast stain It is a special bacteriological stain used to identify acid-fast organisms, mainly Mycobacteria, since its lipid rich cell wall makes it resistant to Gram stain. Can also be used to stain few other bacteria like Nocardia 50) Western Blot test is a confirmatory test used for the diagnosis of a) CMV b) Hepatitis B Virus c) HIV d) Epstein Barr Virus Ans (c)HIV (Text Book Of Microbiology by Ananthanarayan 7th ed page no 592) It is considered as a gold standard test for diagnosis of HIV.

DENTAL QUEST Your Quest For The success In Post Graduate Dental Entrance !

DENTAL

QUEST

KCET -2011 82

51) Name the fungi which are ovoid or spherical in morphology, produces budding cells and pseudohyphae in culture media and in human tissues. a) Histoplasma capsulatum b) Candida albicans c) Blastomyces dermatitidis d) Paracoccidioides brasiliensis Ans (b)Candida albicans (Text Book Of Microbiology by Ananthanarayan 7th ed page no 616) 52) Which of the following organisms does NOT react with gram's stain ? a) Actinomyces israelii b) Candida albicans c) Mycobacterium tuberculosis d) Streptococcus mutans Ans (c)Mycobacterium tuberculosis (Text Book Of Microbiology by Ananthanarayan 7th ed page no 351 ) Its a known fact that gram stain in not taken properly by M Tuberculi. It needs Acid fast stain for its identification.

K C E T 2 0 1 1

ORAL PATHOLOGY
53) Pellagra is seen in deficiency of a) Riboflavin b) Thiamine c) Niacin d) Pyridoxine Ans (c)Niacin (Shafer's textbook of oral pathology 5t ed page no 889-890) Pellagra is a vitamin deficiency disease most commonly caused by a chronic lack of niacin (vitamin B3) in the diet. It can be caused by decreased intake of niacin or tryptophan(required for niacin synthesis i.e. 60 mg of tryptophan for 1 mg of nicotinic acid). Pellagra is classically described by "the four D's": diarrhea, dermatitis, dementia and death. In pellagra epithelial changes followed by characteristic skin rash in areas exposed to sunlight especially in neck region are called CASALS NECKLACE. 54) Decrease in salivary secretion is known as a) Aspahaxia b) Xerostomia c) Aerophagia d) Achalasia Ans (b) Xerostomia (Shafer's textbook of oral pathology 5t ed page no 48-49) XEROSTOMIA (Dry mouth) :- is not a disease but can be a symptom of many diseases. Etiology:Temporary causes 1. Psychological 2. Calculus in dust 3. Sialoadenitis 4. Certain drugs anticholinergics Permanent causes 1. Salivary gland aplasia 2. Sjogrens syndrome 3. Syetemic disorders like diabetes, parkinsons disease,cystic fibrosis and

like and

DENTAL QUEST Your Quest For The success In Post Graduate Dental Entrance !

DENTAL

QUEST

KCET -2011 83

sympathomimimetics.

sarcoidosis. 4. Radiotherapy

55) Shingles is caused by a) Herpes Simplex Virus b) Varicella Zoster Virus c) Herpes Zoster Virus d) Coxsackie Virus Ans (b)Varicella zoster virus (Shafer's textbook of oral pathology 5t ed page no 480) HERPES ZOSTER (shingles, zona): Acute infectious of viral disease characterized by inflammation of dorsal root ganglia or extra medullary cranial nerve ganglia, associated with vesicular eruptions of the skin or mucous membranes in areas supplied by affected sensory nerves. Virus causing this disease is same as that of varicella, or chickenpox i.e. the V-Z virus. Herpes zoster is caused by reactivation of the latent V-Z virus which has been acquired during previous attack of chickenpox. Primary infection by V-Z virus results in chickenpox while recurrent infection results in herpes zoster . REMEMBER: - This infection is typically unilateral and dermatropic in distribution. Also this infection is non-transmissible to animals like herpes simplex virus infections are. 56) Presence of Bence Jones Proteins is characteristic of a) Non-Hodgkins Lymphoma b) Burkitt's Lymphoma c) Multiple Myeloma d) Chronic Lymphocytic Leukemia Ans (c)Multiple myeloma (Shafer's textbook of oral pathology 5t ed page no 261) MULTIPLE MYELOMA:-

K C E T 2 0 1 1


o o

Most common primary neoplasm of skeletal system. Caused by malignancy of plasma cells which are a subset of B-cells.

Clinical Features:Disease of older people (60-65 years) Diffuse disease of bone marrow with 90% of cases having osseous involvement. Oral manifestations: - 95% of cases occurs in mandibular region with ramus, angle and molar region as most frequent sites.

Radiographic features:-numerous sharply punched out areas that varies in size from a few mm to a cm or more in diameter and no peripheral bone formation. Lab findings: Hyperglobulinemia(monoclonal gammopathy) with reversal of albumin-globulin ratio Total serum protein to a level of 8-16 gms percent. Bence Jones Proteins are present in urine of 60-85% which is a protein that coagulates when the urine is heated to 40-60 degree Celsius and then reappears when the urine is cooled. It is also found in some cases of leukemia and polycythemia. The diagnostic finding is monoclonal hypergammaglobulinemia. Igg myeloma is most common followed by IgA myeloma.

There is pain, swelling, expansion of jaws, numbness and mobility of teeth.

DENTAL QUEST Your Quest For The success In Post Graduate Dental Entrance !

DENTAL

QUEST

KCET -2011 84

Hypercalcemia (due to increased bone destruction),hyperuricemia(because of elevated cell turnover)increased ESR and elevated alk.phosphatase levels.

Histologically: - Cartwheel OR Checkerboardpattern

57) General paresis in Neurosyphilis is a feature of a) Primary stage of syphilis b) Latent phase of syphilis c) Secondary stage of syphilis d) Tertiary stage of syphilis Ans (d)Tertiary stage of syphilis (Shafer's textbook of oral pathology 5t ed page no 452) 58) The break-up time test is performed to evaluate the function of a) Salivary flow b) Lacrimal gland c) Thyroid gland d) Blood flow And (b)Lacrimal gland (Burkitts textbook of oral medicine 8th ed 195, you wont find this in new edition)

K C E T 2 0 1 1

The break-up time is the interval between a complete blink and the appearance of dry spot on the cornea which is prolonged in Sjogrens syndrome patient. It is used to evaluate the lacrimal gland function along with the Schirmers test and quantitative Rose Bengal dye test. 59) Hemophilia B is caused due to deficiency of clotting factor a) IX b) VII c) VIII d) X Ans (a)IX (Shafer's textbook of oral pathology 5t ed page no 1082) Type Hemophilia A Hemophilia B Hemophilia C clotting factor deficiency plasma thromboplastinogen(antihemophilic globulin,AHG,factor 8) plasma thromboplastin component (PTC,factor 9) plasma thromboplastin antecedent (PTA,factor 11)

The genes for factor 8 and 9 are located on long arm of X Chromosome in bands q28 and q27 respectively. Classification of hemophilia according to clinical severity:Classification Mild Moderate Severe factor activity in percent > 5 15 <1 cause of hemorrhage Major trauma or surgery Mild to moderate trauma Spontaneous hemarthrosis , soft Tissue bleeding

60) a) b)

Kveim test is done for Aphthous ulcer Sarcoidosis

DENTAL QUEST Your Quest For The success In Post Graduate Dental Entrance !

DENTAL
c) d)

QUEST

KCET -2011 85

Tuberculosis Herpes simplex

Ans (b)Sarcoidosis (Shafers textbook of oral pathology 5t ed page no 924) SARCOIDOSIS (Boecks sarcoid,besnier-boeck-schaumann disease) Is a multi-system granulomatous disease of unknown origin characterized by formation of uniform, discrete, compact, non-caseating epitheloid granulomas. Kveim-siltzbach test is an intracutaneous test for diagnosis of sarcoidosis which utilizes a suspension of human known sarcoidal tissue as the test agent. It is a highly specific test with very few falsepositive results. 61) Kissing disease is also known as a) scarlet fever b) acute herpetic gingivostomatitis c) glandular fever d) rubella Ans (c) Glandular fever (Shafer's textbook of oral pathology 5t ed page no 1065) INFECTIOUS MONONUCLEOSIS (GLANDULAR FEVER): Described by Sprunt and Evans in 1920. The term glandular fever was first used in 1889 and was termed drusenfieber.. Caused by EB Virus Transmitted via intimate contact with body secretions, mainly oro-pharyngeal secretions and one means is thought to be via deep kissing or intimate oral exchange of saliva and hence also called as kissing disease. Clinical features: Fever, sore throat, headache, chills cough, lymphadenopathy (bilateral and symmetrical). Splenomegaly and hepatitis Cervical lymphadenopathy occurs first Disease occurs in age group of 15-20 year group. Pharyngitis and tonsillitis also common. Oral manifestations: Acute gingivitis and stomatitis Appearance of white or gray membrane and palatal petichiae and sometimes oral ulcers. Lab finding:-Normal titer of agglutinin and hemolysins in human blood against sheep red blood cells does not exceed 1:8. In infectious mononucleosis this titer raises to 1:4096 which is referred to as positive Paul-Bunnel test which is a characteristic and pathognomic test of this disease. One of the specific tests is Monospot test in which there is agglutination of horse RBCs on exposure to EB virus heterophile antibodies.

K C E T 2 0 1 1

62)

a) b) c) d)

An 18 year old lady presents with severe sore throat, fever and malaise. She has cervical lymphadenopathy, splenomegaly and scattered petichiae on the soft palate with enlarged tonsils covered with white exudates. Which of the following investigations is most likely io help guide your management ? Fine needle aspiration of a lymph node Hepatitis B surface antigen Heterophil antibody test HIV test

DENTAL QUEST Your Quest For The success In Post Graduate Dental Entrance !

DENTAL

QUEST

KCET -2011 86

Ans (c) Heterophil antibody test (Shafer's textbook of oral pathology 5t ed page no 1065-1068) This is a case of glandular fever, for more SEE EXPLANATION ABOVE 63) Garre's Sclerosing Osteomyelitis is also known as a) Proliferative Periostitis b) Infantile Osteomyelitis c) Chronic sclerosing Osteomyelitis d) Necrosis Ans (a) Proliferative periostitis (Shafer's textbook of oral pathology 5t ed page no 691) CHRONIC OSTEOMYLETIS WITH PROLIFERATIVE suppurative sclerosing osteitis,periostitis ossificans) PERIOSTITIS( Garres chronic non

K C E T 2 0 1 1

Focal and gross thickening of periosteum with peripheral reactive bone formation(i.e. neoperiostitis) caused due to mild irritation or infection. Clinical features Occurs in young adults of age 25 years or less. Most frequently involved area is anterior surface of tibia. In jaws it occurs in mandible in molar and bicuspid region. Radiographic picture:- classical onion skin appearance due to cortical layer duplication. 64) Small palpable mass elevated above the epithelial surface is a) Papule b) Macule c) Plaque d) Vesicle Ans (a)Papule (Burkitt oral medicine 10th ed page no 51) Macules. Well-circumscribed, flat lesions Change in colour as red in presence of vascular lesions or inflammation, or pigmented due to the presence of melanin, hemosiderin, and drugs Papules. Solid lesions raised above the skin surface Smaller than 1 cm in diameter Plaques. Solid raised lesions that are over 1 cm in diameter; Are actually large papules. Nodules. Lesions are present deep in the dermis, epidermis can be easily moved over them. Vesicles. Are Elevated blisters containing clear fluid and are under 1 cm in diameter. Bullae. Over 1 cm in diameter. Erosions. Moist red lesions often caused by the rupture of vesicles or bullae as well as trauma. Pustules. Raised lesions containing purulent material. Purpura. Reddish to purple flat lesions caused by blood from vessels leaking into the subcutaneous tissue. Classified by size as

DENTAL QUEST Your Quest For The success In Post Graduate Dental Entrance !

DENTAL
o o

QUEST

KCET -2011 87

Purpuric lesions 1 to 2 mm in diameter. Larger purpuric lesions are called ecchymoses.

65) A 60 year old male presents with a 2 day history of a severe left ear ache with a burning sensation in the ear and loss of taste. There is left sided weakness of both upper and lower facial muscles. Facial sensation is normal. A few vesicles are seen in the pharynx. What is the most likely diagnosis ? a) Bell's palsy b) Acoustic neuroma c) Diphtheria d) Ramsay Hunt syndrome Ans (d)Ramsay hunt syndrome (Shafer's textbook of oral pathology 5th ed page no. 482) Ramsay Hunt Syndrome Etiology- Special form of herpes zoster infection of geniculate ganglion with involvement of external ear and oral mucosa Signs and symptoms Facial paralysis as well as pain of external auditory meatus and pinna of ear Vesicular eruption of oral cavity and oropharynx ,Hoarseness of voice, Tinnitus Vertigo Vesicular eruptions are always unilateral and dermatomic in distribution Herpes zoster cannot be transmitted to animals unlike herpes simplex Diagnosis- Tzanck test to find multinucleated giant cells in cytologic smears

K C E T 2 0 1 1

66)

Small round superficial erosions in the mouth which coalesce to form the so called snail track ulcers are seen in? a) Primary syphilis b) Secondary syphilis c) Tertiary syphilis d) Congenital syphilis Ans (b)Secondary syphilis (A Concise Textbook of Surgery by S. Das 4th ed page no 586 ) Acquired syphilis Etiology Treponema pallidum, (spirochaete), 615 m in length. Transmission directed by direct contact with a surface lesion Types of acquired syphilles primary, secondary tertiary syphilis

Lesions are only infective in its first 2 years of latency

Clinical features Primary syphilis. Hunterian chancre:- At primary sore or at the site of entry of the treponemes in about 34 weeks Shallow, indurated, painless, non-bleeding ulcer, usually single, oval or round, with a raised hyperaemic margin, often extending into a dusky red oedema. shotty enlargement of lymph nodes which are discrete,painless and rubbery in consistency.

DENTAL QUEST Your Quest For The success In Post Graduate Dental Entrance !

DENTAL

QUEST

KCET -2011 88

Secondary syphilis. In 612 weeks, Commonest sign -a dull red or copperyrash or Papules on contiguous moist sites which enlarge to form condylomata lata, In mouth Small, round, superficial erosions may occur where they coalesce to form the socalled snail-track ulcers. Latent syphilis- seen in untreated secondary stage and lasts from 2 years to a lifetime. No clinical signs but the serum tests are positive. Late syphilis (Tertiary syphilis). Seen 515 years after infection, 10 per cent will develop neurosyphilis, 1012 per cent cardiovascular syphilis Gumma - syphilitic hypersensitivity reaction made up of granulation tissue with presence of central necrosis, an ulcer with punched-out margin with a wash-leather bas. Leaves a silvery tissue-paper scar on healing Diagnosis. Dark-field microscopy. The serum tests only turn positive after 1090 days (usually 35 weeks) after the appearance of the chancre. Serological tests for treponemal diseases are of two types. 1. Nonspecific (syn. reagin, nontreponemal, lipoidal antigen tests) cardiolipin Wasserman (WR), Kahn Meinicke, Venereal Disease Research Laboratory (VDRL) slide test. (best test) Group-specific tests (treponemal antigen tests). Reiters protein complement fixation test (RPCF), Treponema pallidum haemagglutination assay (TPHA), Absorbed fluorescent antibody test (FTA Abs) Treponema pallidum immobilisation test (TPI).

K C E T 2 0 1 1

2.

Genrally VDRL and TPHA are done first. If positive then FTA is done to confirm. In primary syphilis, the usual order of conversion is ETA, VDRL with rising titre and TPHA Once after the treatment course , the VDRL usually reverts to negative in up to 6 months, the FTA in 70 per cent OF CASES, but the TPHA hardly ever Treatment High cure rate is achieved in early syphilis with intramuscular procaine penicillin G 1.2 g daily for 15 days. In penicillin-allergic patients, tetracycline and erythromycin are alternatives. The best treatment is doxycycline 100 mg twice daily for the equivalent period. JanischHerxheimer reaction. 6 hours after the first injection, 60 per cent of early syphilitics will show pyrexia, malaise and presence of rigors lasting for a few hours. 67) Treatment of choice for basal cell carcinoma of skin is?

DENTAL QUEST Your Quest For The success In Post Graduate Dental Entrance !

DENTAL

QUEST

KCET -2011 89

a) Surgical excision b) Radiotherapy c) Chemotherapy d) Laser surgery Ans (a)Surgical excision (Bailey and Love's Short Practice of Surgery 23rd ed page no 181) Basal cell carcinoma (Rodent ulcer) More common in males 85% lesions occur in head and neck region.

K C E T 2 0 1 1

Origin - Pluripotential epithelial cells of the epidermis and hair follicles. It is localy invasive and grows slowly,penetrate deeper tissues hence named as rodent Histologic types Nodular: 5054 % Superficial: 911 % Cystic: 48 % Pigmented: 6 % Morpheic: 2 %

Treatment Surgical excision the treatment of choice and gives 85 to 95 per cent of cure rate Radiotherapy- 92 % cure rate. Mobs micrographic surgery (chemosurgery) for recurrent cases or lesions in the difficult areas or those with indistinct borders 68) The cyst which is associated with a non-vital tooth is a) Stafne bone cyst b) Solitary bone cyst c) Radicular cyst d) Globulo-maxillary cyst Ans (c)Radicular cyst ( Shafers textbook of oral pathology 5t ed page no. 376) Radicular cyst ( Periapical cyst, Apical Periodontal cyst, Root end cyst) Most common odontogenic cyst(60%) Associated with infected tooth with necrotic pulp Toxin stimulate Epithelial rests of Malassez to form cyst. A periapical pocket cyst or bay cyst - an apical inflammatory cyst that contains a sac-like, epithelium-lined cavity that is open to and continuous with the root canal space. True apical cysts are located within the periapical granuloma with no connection between their cavity and that of the root canal space. More than half of the apical cysts are true apical cysts while the remainder are of the pocket cyst variety. Clinical features Most of the time asymptomatic Painful tooth with sensitivity to percussion Expansion of cortical plate Radiographic feature Radiolucency surrounded by radiopaque line , size more than 2 cm Histologic feature Cart-wheel arrangement of nuclear chromatin and presence of Rushton body or Hyaline body- liner arc shaped , amorphous eosinophilic bodies brittle in nature.

DENTAL QUEST Your Quest For The success In Post Graduate Dental Entrance !

DENTAL

QUEST

KCET -2011 90

69) Cylindroma is another name for ? a) Adenoid cystic carcinoma b) Adenolymphoma c) Mucoepidermoid carcinoma d) Pleomorphic Adenoma Ans (a)Adenoid cystic carcinoma ( Shafers textbook of oral pathology 5t ed page no. 330) Different names Salivary Gland tumors Pleomorphic adenoma- Mixed tumor Warthins Tumor- Papillary Cystadenoma lymphomatosuma, Adenolymphoma Adenoid Cystic Carcinoma- Cylindroma,, Adenocystic carcinoma Acinic cell carcinoma- adenocarcinoma Odontogenic cyst and tumors

K C E T 2 0 1 1

70)

Dentigerous cyst- Follicular cyst Dental Lamina cyst of the new born - Gingival cyst of new born, Bhones nodules, Lateral periodontal cyst- Botryoid odontogenic cyst Calcifying Odontogenic cyst- Gorlin Cyst Radicular cyst - Periapical cyst, Apical Periodontal cyst, Root end cyst Ameloblastoma- Adamantinoma, Adamantoblastoma, Multiocular cyst Calcifying epithelial odontogenic tumor- Pindborg Tumor Adenomatoid Odontogenic tumoradenoameloblastoma, Ameloblastic adenomatoid tumor Periapical Cemento-osseous dysplasia occurs frequently in the? a) Mandibular posterior region b) Mandibular anterior region c) Maxillary posterior region d) Maxillary anterior region Ans (b)Mandibular anterior region (Text book of oral and maxillofacial pathology Neville 3rd ed page no 641) Periapical cemento-osseous dysplasia Predominantly involves the periapical region of the anterior mandible. Multiple foci are more common More in female Teeth associated are vital Asymptomatic Radiographic feature Mixed radiolucent and radiopaque appearance. End stage shows circumscribed dense calcification which is surrounded by a narrow radiolucent rim. Intact PDL space present

71)

A developmental alteration where a single tooth bud divides incompletely resulting in the formation of a tooth with a bifid crown and a single pulp chamber is called? a) Fusion b) Concrescence

DENTAL QUEST Your Quest For The success In Post Graduate Dental Entrance !

DENTAL

QUEST

KCET -2011 91

c) Gemination d) Dens Evaginatus Ans (c)Gemination (Shafer's textbook of oral pathology 5t ed page no 54) Fusion Union of two normally separated tooth germ If occurs before calcification then single large tooth formed In true fusion dentin is always confluent Gemination An attempt at division of a single tooth germ by invagination which results in incomplete formation of two teeth Single root can root canal and may have single or separated crown Twinning is when a normal and a supernumerary tooth is formed Concrescence Fusion after root formation Only fusion due to deposition of cementum between two root

K C E T 2 0 1 1

MEDICINE
72) A 70 year old male is admitted to the hospital with a left hemiparesis and altered consciousness. He is on aspirin 75mg OD, atrovostatin 10 mg OD and glibenclamide 15mg daily. His wife says he has been unwell for a couple of days and he has been off food. He has however, still been giving him all his medications. Which of the following tests is going to be most useful in finding a immediately reversible cause for his symptoms a) Blood glucose levels b) CT brain c) ECG d) Troponin levels Ans (a)Blood glucose levels (K.D. Tripathi Essentials of Medical Pharmacology 5th ed page no 248) In the above case the patient is on Glibenclamide which is a second generation sulfonylureas, an oral antidiabetic drug. The most common adverse effect with these drugs is said to be hypoglycemia which leads to altered consciousness. And in the above case the patient has been taking aspirin which is contraindicated in both diabetic patients and those taking sulfonylureas as it increases the toxicity of these drugs by its displacement from plasma proteins. Also he has been off from food for few days which may lead to hypoglycemia easily but it surely can be reversed. Hence the Blood glucose level should be estimated first. Oral Hypoglycemic drugs Sulfonylureas First generation Tolbutamide Chlorpropamide Second generation Gilbenclamide Glipizide Gliclazide Glimepiride Biguanides Phenformin Repaglinide Metformin Meglitinide Analogues Nateglinide Thiazolidinediones

DENTAL QUEST Your Quest For The success In Post Graduate Dental Entrance !

DENTAL

QUEST

KCET -2011 92

Rosinglitazone

Pioglitazone

73) A 50 year old female with end stage renal disease (ESRD), develops pulmonary tuberculosis. Which one of the following drugs should be used in a reduced dose? a) Rifampicin b) Isoniazide c) Pyrazamide d) Ethambutol Ans (d)Ethambutol (K.D. Tripathi Essentials of Medical Pharamcology 5th ed page no 699-701) Side effects of ethambutol is hyperuricemia due to its interference with urate excretion. Hence it should be used in reduced doses in end stage renal diseases as its excretion is manly dependent on glomerular filtration. In case of reduced renal function, the main side effect optic neuritits will occur rapidly as this adverse effect is dose dependent.

K C E T 2 0 1 1

74)

A young man presents :with jaundice and itching. His viral serology showed active Hepatitis E infection. Which of the following is true of Hepatitis E ? a) Chronic Hepatitis always occurs b) Hepatitis E can be prevented by passive immunization c) Transmission is by feco-oral route d) Co infection with hepatitis B is required for pathogenicity Ans (c)Transmission is by feco-oral route (Text Book Of Microbiology by Ananthanarayan 7th ed page no 559) TYPE Mode infection Age of A 27 nm RNA Picorvirus Fecal-oral Children 15-45 days Nil ---B 47 nm DNA Hepdnavirus Percutaneous sexual, vertical Any age 30-180 days Common Present after neonatal infection Common Ig and Vaccine Insidious and occasionally serious C 30-60 nm RNA Flavivirus Percutaneous Adults 15-160 days Present Present after neonatal infection Probably common Nil Insidious and moderate D 35-37 nm RNA E 32-34 nm RNA

Percutaneous Fecal-oral Adults 30-180 days Nil ( only with HBV) ---Young adult 15-60 days Nil ----

Incubation period Carrier state Oncogenecity

Prevalence Prophylaxis Severity of illness

Common Ig and Vaccine Acute mild

Endemic HBV vaccine Insidious and occasiona lly serious

Only in developing countries Nil Mild but sever in pregnancy

75)
a) b) c) d)

Oxygen as an emergency drug is contraindicated in? Anaphylaxis Asthma Hyperventilation COPD

DENTAL QUEST Your Quest For The success In Post Graduate Dental Entrance !

DENTAL

QUEST

KCET -2011 93

Ans (c)Hyperventilation ( Hand book of Medical Emergency in Dental Office 2nd Ed Page no 153) Hyperventilation is due to decreased Co2 content in the blood. Administration of oxygen will again decrease the content of Co2. Hence administration of oxygen is contraindicated. 76) You are managing a cardiac arrest. Which drug is to be given for ventricular fibrillation after unsuccessful defibrillation ? a) 1 mg Atropin b) 1 mg Epinephrine c) 1 mg/kg NaHC03 d) 1 mg/kg Lidocaine Ans (b)1mg epinephrine (DAVIDSON TEXTBOOK OF MEDICINE 18TH EDITION PG NU 557,FIG 18.33) Management of Cardiac Arrest Cardiac Arrest do pericardial thump- do BLS algorithm- use defibrillator- Perform CPR- it still not responsive inject 1mg/3m in Epinephrine 77) Ivy technique is a test done to measure the a) Bleeding time b) Capillary fragility c) Prothrombin time d) Platelet survival time Ans (a) Bleeding time (TEXTBOOK OF MEDICAL PHYSIOLOGY BY BIJLANI,3RD EDITION) 78) Mantoux test is done for the diagnosis of a) Syphilis b) Cat scratch c) Tuberculosis d) Tetanus Ans (c) Tuberculosis (Davidsons principles and practice of medicine 19th ED page no. 537) The Mantoux test (also known as the Mantoux screening test, Tuberculin Sensitivity Test, Pirquet test, or PPD test for Purified Protein Derivative) is a diagnostic tool for tuberculosis apart from Heaf test. The Heaf test is a tuberculin skin test formerly used in the United Kingdom, but discontinued in 2005.The equivalent Mantoux test positive levels done with 10 TU (0.1 mL 100 TU/mL, 1:1000) are * <5 mm induration (Heaf 0-1) * 514/15 mm induration (Heaf 2) * >14/15 mm induration (Heaf 3-4) In Mantoux test a dose of 10 Tuberculin units is injected intradermally (between the layers of dermis) and read 48 to 72 hours later. This intradermal injection is termed the mantoux technique. The test is positive when induration is of 5-14mm(equivalent to heaf grade 2)or > 15mm(heaf grade 3-4). 79) A patient presents with jaundice. Physical examination reveals an enlarged nodular liver. CT of the abdomen shows a cirrhotic liver with a large mass. CT guided biopsy of the mass demonstrates a malignant tumor derived from hepatic parenchymal cells. Infection with which of the following viruses would most directly related to the development of this tumor? a) Epstein-Barr virus b) Human Herpes virus type A c) Human T-lymphocyte virus d) Hepatitis B virus

K C E T 2 0 1 1

DENTAL QUEST Your Quest For The success In Post Graduate Dental Entrance !

DENTAL

QUEST

KCET -2011 94

Ans (d) Hepatitis B virus (Robbins & Cotran Pathologic Basis of Disease, 7th ed page no page no 327 and Harrison Principles Of Internal Medicine 17th ed Page no 580-583) 10% of Hepatocellular carcinoma (HCC) is likely to develop cirrhosis Etiology Chemical Carcinogens - a product of Aspergillus fungus, called aflatoxin B1. Hepatitis - strong association between chronic hepatitis B carrier rates and increased incidence of HCC. Other Etiologic Conditions. 7585% association of HCC with underlying cirrhosis has long been recognized, typically with macronodular cirrhosis Symptom and sign Abdominal pain Jaundice Hepatomegaly Routine CT scan screening of known cirrhosis symptoms (ankle swelling, abdominal bloating, increased girth, pruritus, GI bleed) Male: Female 3:1 Tumour Marker Alfa-Fetoprotein (AFP) is a serum tumor marker in HCC des--carboxy prothrombin (DCP), a protein induced by vitamin K absence (PIVKA-2). Treatment Stage I and II Surgical resection Local ablation (thermal or radiofrequency) Local injection therapies (ethanol or acetic acid). Liver transplant Stage III and IV- Hepatectomy 80) A 30 year old known SLE female presents with ankle swelling. Her BP is 170/100 mmllg and she has marked lower limb edema. Urine dip stick discloses 4+ proteins with no hematuria. Plasma creatinine is 1.4 mg/dl, albumin is 1.5g/dl and urine protein excretion rate was 6 gms for 24 hour. What is the cause of this presentation ? a) Amyloidosis b) Class II lupus nephritis c) Class V lupus nephritis d) Pericardial effusion Ans (c)Class V lupus nephritis ( Harrison Principal Of Internal Medicine 14th ed Page no 1548 ) Class V lupus Nephritis is associated with neprotic syndrome in 90% cases. The examiner has just put the symptoms of Nephrotic syndrome with class V. 81) If during blood transfusion, transfusion reaction develops in the form of incompatibility , the first thing to be done is? a) Stop the transfusion b) Inj Hydrocortisone to be given stat c) Inj Chlorpheneramine maleate to be given at stat d) Inj Frusemide to be given Ans (a) Stop the transfusion ( Davidson's Principles and Practice of Medicine 19th ed page no.915) Steps in management of incompatible blood transfusion 1. Stop blood transfusion 2. If Urticaria present then give Chlorphenamine(Chlorpheneramine) 10mg slowly. If no urticarial then, 3. Suspect ABO incompatibility give Furosemide to maintain urine output.

K C E T 2 0 1 1

DENTAL QUEST Your Quest For The success In Post Graduate Dental Entrance !

DENTAL

QUEST

KCET -2011 95

4. If severe allergic reaction present then give Chlorpheneramine 10mg iv. or salbutamol nebulizer, 5. If no allergy but sever hypertension present- give 0.5ml of adrenaline 1 in 1000/0.5 mg i.v 6. If no hemolytic reaction and there is presence of dyspnea/ hypotension then give 100% oxygen to cure Transfusion related lung injury (TRALI) 82) A 30 year old lady, previous rheumatic heart disease presents with 3 month history of fatigue, weight loss, night sweats and dyspnoea on exertion. She is pyrexial with evidence of mitral regurgitation and splinter hemorrhages. ECHO confirms vegetations, blood cultures arc taken and a diagnosis of infective endocarditis is made. What is the most likely infecting organism in this case ? a) Streptococcuss viridins b) Staphylococcus aureus c) Staphylococcus epidcrmidis d) Entcrococcus sp Ans (a)Streptococcus viridians ( Davidsons Principles and Practice of medicine 19th edition page no.464) Guys. The diagnosis is already given right . Streptococcus viridians is the most common microorganism found in the case having a history of previous Rheumatic heart disease.

K C E T 2 0 1 1

83)

The stimulus for parathyroid hyperplasia in secondary hyperparathyroidism is a) Chronic hypocalcaemia b) Chronic hypocalcaemia c) Chronic hypomagnaesemia d) Chronic hypermagnaesemia Ans (b)Chronic hypocalcaemia (Davidson's Principles and Practice of Medicine 19th ed page no.718) Secondary hyperparathyroidism occurs when there is increased parathyroid hormone secretion to compensate hypocalcaemia and associated with hyperplasia of all parathyroid tissue.

GENERAL SURGERY
84)
According to the "Rule of Nine" in burns, what is the percentage of burns if both the upper limbs and lower limbs are involved? a) 24% b) 36% c) 54% d) 72% Ans (c) 54% (A Concise Textbook of Surgery by S. Das 4th ed page no 52) One can estimate the body surface area on an adult that has burned by using multiples of 9. An adult who has been burned, the percent of the body involved can be calculated as follows:

Head = 9% Chest (front) = 9% Abdomen (front) =


9%

Upper/mid/low back
and buttocks = 18% Each arm = 9% Each palm = 1%

Groin = 1% Each leg = 18% total


(front = 9%, back = 9%)

As an example, if both legs (18% x 2 = 36%), the groin (1%) and the front chest and abdomen were burned, this would involve 55% of the body adult with more than 20 per cent of the body surface involved or a child with more than 10 per cent of body surface area involved will require intravenous fluid replacement If the age and percentage added and score 100 is found then the burn is likely to be fatal. A child may survive a large burn, but even a small burn in an elderly patient is potentially fatal.

DENTAL QUEST Your Quest For The success In Post Graduate Dental Entrance !

DENTAL 85)

QUEST

KCET -2011 96

Commonest location of a thyroglossal cyst is? a) Floor of mouth b) Suprahyoid c) Subhyoid d) In the region of thyroid cartilage Ans (c)Subhyoid (A Concise Textbook of Surgery by S. Das 4th ed page no 627) According to its frequency of occurrence the sites are as follows Subhyoid position- most common In the region of thyroid cartilage where it is mostly found on left side Suprahyoid The level of cricoid cartilage Floor of mouth Below foramen caecum- least common

K C E T 2 0 1 1

86)
a) b) c) d)

Pleomorphic adenomas arising from the minor salivary glands can be treated with ? Local excision with 2mm margin Local excision with 5mm margin Radiotherapy Chemotherapy

Ans (b) Local excision with 5mm margin (Bailey Surgical Textbook 23rd ed page no 660) Pleomorphic Adenoma (Mixed tumor , encranchioma,endothelioma,endoclavoma, enchondroma) Is a benign neoplasm consisting of cells exhibiting the ability to differentiate to epithelial cells(ductal and nonductul) and mesenchymal cells(chondroid, myxoid and osseous).Its not a mixed tumor in true sense. Most common salivary gland tumor. Parotid gland most common site(90%) in which it mostly involves lower pole of superficial lobe 8% cases involve minor gland. In most cases palatal gland are involved, More common in females Also seen as dumble tumor Histogenesis Resereve cells and intercalated cells are responsible for the formation . Cytogenetic abnormality involving chromosome no 12q13-15. Presence of Hyaline cells and myoepithelial cells Varientso Principally Myxoid o Predominantly cellular o Myxoid and cellular together o Extremely cellular Treatment For Parotid gland either superficial or total Parotidectomy according to the location of the tumour. If no facial nerve palsy (indicating a malignant Tumour) present then facial nerve is always preserved. Submandibular gland treatment is always by excision of the gland. Pleomorphic adenomas arising in the minor salivary glands can be treated by local excision with a 5-mm margin..

87)
a) b) c)

Acute infection of bacteria which affects the nervous system? Syphilis Meliodosis Tetanus

DENTAL QUEST Your Quest For The success In Post Graduate Dental Entrance !

DENTAL

QUEST

KCET -2011 97

d) Tularemia And (c)Tetanus (Shafers textbook of oral pathology 5t ed page no.449 ) Tetanus(Lock Jaw) Acute infection of nervous system characterized by intense activity of neurons which further results in muscle spasm. Caused by Clostridium Tetany Clinical features Incubation period 6-10 days Generalized tetanus o Lock jaw or Trismus due to spams of Masseter as the first symptom. o Dysphagia , stiffness or pain in neck and shoulders o Risus scardonious due to grimace or sneer of facial muscle o Opisthotonous arched back Cephalic tetanus Trismus and facial palsy together Local tetanus 88) Koplik's spots are commonly seen in a) Measles b) Rubella c) Chicken pox d) Small pox Ans (a) Measles (Shafers textbook of oral pathology 5t ed page no. 476) Measles (Robeola, Morbilli) o Caused by Paramyxovirus an RNA virus o Portal of entry through direct contact or droplet infection or through respiratory tract. o Upon entry it reaches Reticuloendotelium system and there by involves skin respiratory tract and other systems along with suppression of immunity which mostly involves monocytes. o Incubation period 8-10 days o Intraoral lesions termed as Koplik spots and seen in 97% cases which disappear once the onset of rash seen. They are small flecks with bright red margin with a central bluish core o Histopathologially Warthinfinkildey gaint cells seen

K C E T 2 0 1 1

89)

A form of fibrous dysplasia occurring in children is termed as? a) Monostotic form b) Polyostotic form c) Craniofacial form d) Cherubism And (d)Cherubism (Shafer's textbook of oral pathology 5t ed page no. 979) Types of Fibrous Dysplasia o Monostotic from o Polystotic form Cherubism( Familial Fibrous Dysplasia of Jaw) o Autosomal dominant o Involves more than one quadrant o o Craniofacial Form Cherubism o Stabilizes after growth period(starts by 14 months to 3 years)

DENTAL QUEST Your Quest For The success In Post Graduate Dental Entrance !

DENTAL

QUEST

KCET -2011 98

o Gene 4p16.3 affected Clinical features Cherubic look More in males Mostly bilateral Cervical lymphadenopathy Radiographic Features o Floating tooth syndrome o Ground glass appearance Histologic Feature Eosinophilic cuffing 90) Rh hump is commonly associated with a) Iron deficiency anemia b) Aplastic anemia c) Erythroblastosis fetalis d) Sickle cell anemia Ans (c)Erythroblastosis fetalis ( Shafer's textbook of oral pathology 5t ed page no.1053)

Eye in to heaven appearancea rim of sclera visible below iris Associated with Noonans Syndrome

K C E T 2 0 1 1

Erythroblastosis Fetalis Congenital hemolytic anemia due to Rh incompatibility Oral manifestationso Deposition of blood pigment in the developing teeth enamel or dentin which contains bilirubin. In few cases the enamel hypoplasia can be found in incisal edges of anterior teeth , middle potion of primary canine and first molar crown is present which is termed as Rh hump as termed by Watson 91) Coup de sabre is commonly associated with a) Pemphigus b) Systemic sclerosis c) Erythema multiforme d) Systemic lupus erythematosus Ans (b)Systemic sclerosis ( Shafer's textbook of oral pathology 5t ed page no.1146) Systemic sclerosis- Hidebound disease Systemic connective tissue disorder, consisting of vasomotor disturbances, fibrosis, and atrophy of skin, muscle, and internal organs. Clinical Features Mask like appearance of face Microstomia Skin yellow, gray, ivory white Dysphagia waxy in nature Presence of salivary Brown pigmentation disorder- sjogrens syndome Stiff tongue Crest Syndrome- A variant of Systemic sclerosis Calcinosis cutis Raynauds phenomenon Esophageal dysfunction Sclerodactyly Telangiectasia Morphe- Circumscribed scleroderma Coup de sabre- A liner scleroderma, a bad made up of furrow with elevated ridge on one side. It is also found in Facial Hemiatrophy or Pary-Romberg Syndrome

gland

DENTAL QUEST Your Quest For The success In Post Graduate Dental Entrance !

DENTAL

QUEST

KCET -2011 99

92) Histological appearance of "Abtrofung or dropping off' is commonly seen in? a) Intradermal nevus b) Junctional nevus c) Compound nevus d) Epithelioid nevus Ans (b) Junctional nevus ( Shafer's textbook of oral pathology 5t ed page no.120) Junctional nevus The zone of demarcation between the connective tissue and overlaying epithelium is absent and cells are found crossing the junction and growing down to the connective tissue. This is called as Abtrofung or dropping off effect

K C E T 2 0 1 1

93)

The causative agent For "Oculoglandular syndrome of Parinaud" is ? a) Arachnia propionica b) Bartonella henselae c) Bifidobacterium dentium d) Mycobacterium leprae Ans (b)Bartonella henselae ( Shafer's textbook of oral pathology 5t ed page no. 458) Parinaud's oculoglandular syndrome is the combination of granuloma , and swollen lymph nodes in front of the ear on the same side. Most cases are caused by cat-scratch disease. Etiology of cat scratch disease Bartonella Henselae enters the skin due to scratch or bite of household cat which is just a carrier. Primary lesion- Papule as site of entry After 1- 3 week- regional Lymphadenitis without lymphangioma In Oculoglandular syndrome of Parinaud the lymph nodes become soft and fluctuant due to necrosis and suppuration. The organism is can be identifies by Warthin Starry Silver staining and Brown Hopps gram staining can also be used. Arachnia propionica and Bifidobacterium Dentium- vary rarely found to cause Actinomycosis 94) a) b) c) d) Salivary peroxidase system is known to be effective against Actinomyces viscosus Lactobacillus acidophilus Streptococcus mutans Veillonella parvula

Ans (b) Lactobacillus acidophilus ( Shafer's textbook of oral pathology 5t ed page no. 594) Antibacterial properties of Saliva Lysozyme A hydrolytic enzyme Along with surfactant sodium lauryl sulfate can kill many cariogenic and non cariogenic streptococci Salivary peroxidase system Develops OSCN a hypothiocyanate ion reacts with sulfhydryl compounds of low molecular weight and in activates enzymes of glycolytic pathway

DENTAL QUEST Your Quest For The success In Post Graduate Dental Entrance !

DENTAL

QUEST

KCET -2011 100

Active against Lactobacillus acidophilu, streptococcus cremoris where it prevents lysine and glutamic acid uptake Immunoglobin Mainly IgA in which secretory IgA or SIgA which is around 4mg/ 100 ml in parotid and submandibular and 30mg/100ml higher in minor salivary gland 95) Which of the following is NOT a feature of Hutchinson's triad ? a) Facial nerve paralysis b) Mulberry molars c) Ocular interstitial keratitis d) Vestibulocochlear nerve paralysis Ans (a)Facial nerve paralysis ( Shafer's textbook of oral pathology 5t ed page no.453) Congenital syphilis Vertical transmission Hutchinsons classic triad consist of the following: o Interstitial keratitis: ground glass appearance of the cornea, o Eighth nerve deafness o Hutchinsons teeth: a peg or band-shaped deformity of the upper central incisors, Moons molars (mulberry molars) o Other signs include Saddle nose Perforation of the palate Parietal bossing Collapsed nasal Sabre tibia, Septum Gluttons joints Prevention and treatment. -1.2 million Units of procaine penicillin G given to the mother for 15 days as early as possible

K C E T 2 0 1 1

96)

An example of a sex related disease more commonly seen in males is a) Primary Herpetic gingivostomatitis b) Anorexia c) Early childhood caries d) Hemophilia Ans (d)Hemophilia (Shafer's textbook of oral pathology 5t ed page no 1081) Primary Herpetic gingivostomatitis- Viral disease Anorexia- Eating disorder Early childhood caries- mostly related to diet Hemophilia- X- linked bleeding disorder. Females typically carry the trait, but it is expressed primarily in males.

ORTHODONTICS
97) The first speech sounds produced by developing child is a) Linguo alveolar b) Sibilant c) Linguo dental d) Bilabial Ans (d)Bilabial (Contemporary Orthodontics by Profitt Page no 85) The first speech sounds are the bilabial sounds /m/, /p/, and /b/-which is why an infant's first word is likely to be "mama" or "papa." And later the tongue tip consonants like /t/ and /d/ appear.

DENTAL QUEST Your Quest For The success In Post Graduate Dental Entrance !

DENTAL

QUEST

KCET -2011 101

The sibilant /s/ and /z/ sounds, which require that the tongue tip be placed close to but not against the palate, come later still, and The last speech sound, /r/, which requires precise positioning of the posterior tongue, often is not acquired until age 4 or 5. 98) Surface of NiTi wire is a) rougher than beta titanium wire b) rougher than stainless steel wire c) both of the above d) none of the above Ans (c)Both of the above (Contemporary Orthodontics by Profitt Page no, 378) The surface of NiTi is rougher (because of surface defects and not the quality of polishing) than that of beta-Ti, which in turn is rougher than steel. NiTi has greater surface roughness but beta-Ti has greater frictional resistance and depends on titanium content of an alloy. Beta-Ti, at 80% titanium, has a higher coefficient of friction than NiTi at 50% titanium, and there is greater frictional resistance to sliding with either than with steel. 99) Which of the following is NOT an essential diagnostic aid in orthodontics? a) Case history b) Clinical examination c) Plaster Study casts d) Cephalometric X-rays Ans (d)Cephalometric X-rays ( Balaji text book of orthodontics 3rd ed page no 115-116) ESSENTIAL DIAGNOSTIC AIDS: - (simple, important for all cases , and does not require special equipments) 1. Case history 2. Clinical examination 3. Study models 4. Certain radiographs b) Periapical c) Bite wing d) Panoramic 5. Facial photographs SUPPLEMENTAL DIAGNOSTIC AIDS: - (not essential in all cases) 1. Specialized radiographs b) Cephalometric radiographs c) Occlusal intra-oral films d) Selected lateral jaw views e) Cone shift technique 2. Electromyographic examination of muscle activity. 3. Hand-wrist radiograph to access bone age or maturation age. 4. Endocrine tests 5. Estimation of basal metabolic rate. 6. Diagnostic setup 7. Occlusograms

K C E T 2 0 1 1

100) Catlan's appliance is given for a period NOT more than 6 weeks because it?
a) b) c) Causes attrition of upper anteriors Causes periodontitis of anteriors Causes gingival recession in lower anteriors

DENTAL QUEST Your Quest For The success In Post Graduate Dental Entrance !

DENTAL

QUEST

KCET -2011 102

d) Causes over eruption of posterior teeth Ans (d)Causes over eruption of posterior teeth (Balaji text book of orthodontics 3rd ed page no 429) Catlan's appliance or lower anterior inclined plane: Used to treat maxillary teeth cross-bite. Can be used to treat a single-tooth cross bite or a segment of upper arch in cross-bite. Designed to have a 45 degree angulation that forces maxillary teeth in cross bite to a more labial position. Adequate space should be present in arch for alignment of maxillary teeth in cross-bite before using it Disadvantages:1. Speech problem and dietary restrictions during appliance therapy. 2. If appliance is used for more than 6 weeks it can result in anterior open-bite due to supra eruption of posteriors. 3. Appliance requires frequent recementation. 101) a) b) c) d) Bolton's analysis is used to determine the Proportion of mesiodistal width of the mandibular teeth to that of the basal bone Proportion of mesiodistal width of the maxillary teeth to that of the basal bone Proportion of mesiodistal width of the maxillary teeth to that of the transverse width Proportion of mesiodistal width of the mandibular teeth to that of the maxillary teeth

K C E T 2 0 1 1

Ans (d)Proportion of mesiodistal width of the mandibular teeth to that of the maxillary teeth (Balaji text book of orthodontics 3rd ed page no 178-179) Boltons analysis evaluates the disproportion due to tooth size in mandibular and maxillary teeth. According to it, there exists a ratio between the mesio-distal widths of maxillary and mandibular teeth.

The sum of mesio-distal width of mandibular teeth anterior to the second permanent molars is 91.3% the mesio distal width of the maxillary teeth mesial to the second molars. If the overall ratio is less than 91.3% it indicates maxillary tooth material excess and if the ratio is more than it indicates mandibular tooth material is excess. The sum of mesio-distal width of mandibular anteriors should be 77.2 % the mesio distal width of the maxillary anteriors. If the overall ratio is less than 77.2% it indicates maxillary anterior excess and if the ratio is more than it indicates mandibular anterior excess.

102) Who proposed the "Trajectory theory of bone formation?


a) An anatomist, Meyer b) a mathematician, Cullman c) both a and b d) none of the above Ans (c) Both a and b (Textbook of Bones and cartilage: developmental and evolutionary skeletal biology by Keith Hall 1st ed pg no 410)

Brian

MEYER compared the Trabeculous structure of bone with the stress to which the bone was subjected and he found that the Trabeculae follows lines of stress.in 1866 Meyer was assisted by a Swiss engineer named Cullman in this study. This theory is now termed as WOLFS LAW.

103) Space regaining is indicated in all of the following EXCEPT? a) One or more permanent teeth have been b) Some space in the arch has been lost due to mesial drift of 1M permanent molar
c) Mixed dentition analysis shows that, once the lost space is gained back, there will not be any arch length-tooth material discrepancy d) 1st molars are in end to end relation due to Class II skeletal base and prognathic maxilla

DENTAL QUEST Your Quest For The success In Post Graduate Dental Entrance !

DENTAL
st

QUEST

KCET -2011 103

Ans (d) 1 molars are in end to end relation due to Class II skeletal base and prognathic maxilla. (Moyers handbook of orthodontics 4th edition pg 363-364) Guys one more time after Comedk same year, a straight lift from moyers. Those first three options are the given indication of space regaining. Hence answer is by exclusion. 104) Concept of lingual orthodontic appliance was introduced by? a) Atkinson b) Mershon c) Johnson d) Kurz Ans (d)Kurz (Textbook of Orthodontics By Singh 2nd edition page no 491)

K C E T 2 0 1 1

105) Intermittent forces are applied with the help of?


a) elastics b) headgear c) removable plates d) all of the above Ans (d)All of the above (Contemporary Orthodontics by Profitt Page no 341) Orthodontic force duration is classified by the rate of decay as: Continuous - Force maintained at some appreciable fraction of the original from 1st patient visit to the next Interrupted - Force levels decline to zero between activations. Both continuous and interrupted forces can be produced by fixed appliances that are constantly present. Intermittent- Force levels decline abruptly to zero intermittently, when the orthodontic appliance is removed by the patient or when a fixed appliance is temporarily deactivated and return to the original level some time later. When tooth movement occurs, force levels will decrease as they would with a fixed appliance (i.e., the intermittent force can also become interrupted between adjustments of the appliance). Intermittent forces are produced by all patient-activated appliances, such as removable plates, headgear, and elastics. Forces generated during normal function (chewing, swallowing, speaking, etc.) can be considered as a special case of intermittently applied forces, most of which are not maintained for enough hours per day to have significant effects on the position of the teeth. 106) Which of the following drug is better analgesic for patient who is undergoing orthodontic tooth movement? a) Acetaminophen b) Aspirin c) Ibuforpen d) Diclofenac sodium Ans (a)Acetominophen (Contemporary Orthodontics by Profitt Page no 281) Acetaminophen (Tylenol) is a better analgesic for orthodontic patients than aspirin, ibuprofen, naproxen and similar prostaglandin inhibitors as it acts centrally rather than as a prostaglandin inhibitor. But Acetaminophen is less effective as pain inhibitors.

DENTAL QUEST Your Quest For The success In Post Graduate Dental Entrance !

DENTAL

QUEST

KCET -2011 104

Patients who are not taking prostaglandin inhibitors chronically, there is no effect on initial tooth movement from modest doses over 3-4 days after beginning treatment but on Chronic use these inhibit tooth movement.

107) Myofunctional appliances are indicated in cases with?


a) A class II skeletal malocclusion with a faulty mandible b) A class II skeletal malocclusion with a faulty maxilla c) Appliances identify the fault and correct the malocclusion d) Severe tooth material excess of 14 mm or more Ans (a) A class II skeletal malocclusion with a faulty mandible (Balaji textbook of orthodontics 3rd edition pg no 333) Myofuctional appliances are indicated in moderate to severe class 2 malocclusions with short or retrognathic mandible.

K C E T 2 0 1 1

108) The most important reason for placing an alveolar graft in cleft palate patient is ?
a) Improve pronunciation b) Improve breathing efficiency c) Make eruptive pathway for permanent canine d) Improve feeding efficiency Ans (c) Make eruptive pathway for permanent canine (Contemporary Orthodontics by Profitt Page no, 323-324) Alveolar bone grafts in infancy appear to be contraindicated but placing a bone graft in the alveolar cleft area before the permanent lateral incisors (if present) or permanent canines erupt, is advantageous as it stabilizes the cleft area and creates a healthy environment for the permanent teeth. Ideally, the permanent laterals or canines should erupt through the graft, which means that the best time to place such a graft is between 7 and 10 years. Any necessary alignment of incisors or expansion of posterior segments should be completed before the alveolar grafting. 109) Early mesial shift makes use of a) Incisal liability b) Primate space c) Physiologic spaces d) leeway space of Nance Ans (b)Primate space ( Balaji text book of orthodontics 3rd ed page no 43) Early mesial shift used Primate Spaces Late mesial shift uses Leeway Space

ORAL SURGERY
110) Marx - University of Miami protocol for osteoradio-necrosis is related to?
a) High dose antibiotic therapy b) Hyper baric oxygen therapy c) Both a & b d) None of the above Ans (b)Hyperbaric oxygen therapy ( Text book of Oral and maxillofacial surgery by Neelima Malik 1st ed page no.619)

111) The Orthognathic surgery that is commonly carried out for correction of mandibular
Prognathism is a) Sagittal spilt osteotomy b) Le fort 1 osteotomy

DENTAL QUEST Your Quest For The success In Post Graduate Dental Entrance !

DENTAL
c) d)

QUEST

KCET -2011 105

Vertical ramus osteotomy Genioplasty

ANS (a)Saggital split osteotomy ( Textbook of Oral Surgery by Neelima malik,1st edition pg no 27 ) Bilateral Sagittal split osteotomy is most popular procedure done on mandibular ramus and body which helps for both set back and advancement of mandible with a high cosmetic result without any extra oral scar. It was first described by TRAUNER & OBWEGESER and later modified by DELPONT,HUNSUSK AND EPKER 112) a) b) c) d) Carmalts Gland are Major salivary gland Minor salivary gland (Retromolar) Minor salivary gland (Lingual) Taste Buds

K C E T 2 0 1 1

Ans (b)Minor salivary gland(Retromolar) (Contemporary oral and maxillofacial surgery peterson 435-436)

Tonsillar -Weber's glands Retromolar Carmalt's glands

113) Dilation of the salivary duct secondary to epithelial atrophy as a result of repeated inflammatory or infectious process is a) Sialodochitis b) Sialadeniti c) Sialolithiasis d) Mucocele Ans (a)Sialodochitis (Contemporary oral and maxillofacial surgery peterson 440) 114) Rhytidectomy is a) Removal of skin wrinkles b) Alteration of nasal septum position c) Salivary gland excision d) Cheek augmentation Ans (a)Removal of skin wrinkles ( PETERSON'S PRINCIPLES OF ORAL AND MAXILLOFACIAL SURGERY 2ND ED PAGE NO 1365) Rhytidectomy The goal of facial rejuvenation should be to treating all the components of aging, and giving a patient a younger-appearing face and a long lasting result. Generations of Rhytidectomy I- Subcutaneous dissection only with variable skin undermining II- Subcutaneous dissection + superficial musculo aponeurotic system plication or imbrication III- Subcutaneous dissection + superficial musculoaponeurotic system plication or imbrication + deep mid face section dissection IV- Composite dissection 115) a) b) c) Trans osseous wiring is an example of IMF with osteosynthesis Osteosynthesis without IMF IMF

DENTAL QUEST Your Quest For The success In Post Graduate Dental Entrance !

DENTAL

QUEST

KCET -2011 106

d) None of the above Ans (a) IMF with Osteosynthesis (Williams Maxillofacial Injury 2nd Ed Page no 341) 116) A morbid patient unlikely to survive 24 hours with or without surgery is classified as a) ASA I b) ASA III c) ASAV d) ASA VI Ans (c)ASA V (Malamed's Local Anesthesia5th ed page no 148) The ASA physical status classification system is a system for assessing the fitness of patients before surgery. In 1963 the American Society of Anesthesiologists (ASA) adopted the five-category physical status classification system; a sixth category was later added. These are: 1. A normal healthy patient. 2. A patient with mild systemic disease. 3. A patient with severe systemic disease. 4. A patient with severe systemic disease that is a constant threat to life. 5. A moribund patient who is not expected to survive without the operation. 6. A declared brain-dead patient whose organs are being removed for donor purposes. 117) The 'hanging drop' sign, if visualized in the Water's projection of the traumatized face indicates a) Nasal bone fracture b) Orbital floor fracture c) Zygomatic arch fracture d) Le Fort 1 fracture Ans (b)Orbital floor # ( Kellys Book Of Maxillofacial trauma 5th ed page no 47) Hanging Drop Sign- Smooth convexity facing downwards , suspended from roof of maxillary sinus and made up of extravasated blood as seen in fracture of floor of the orbit on a standard sinus radiograph.

K C E T 2 0 1 1

118) According to Food and Drug administration lactation categories S Refers to?
a) b) c) d) Safe for nursing infant, medication usually compatible with breast feeding Safety in nursing infants unknown; inadequate literature available Potential for significant effects on nursing infants medication should be given with caution Not safe for nursing infants; medication contraindicated or requires cessation of breast feeding

Ans (c) Potential for significant effects on nursing infants medication should be given with caution (Malamed's Local Anesthesia 5th ed page no 146) Food and Drug Administration Lactation Categories Lactation Category S S? S* NS Definition Safe for nursing infant , and medication compatible with breast feeding Safety in nursing infants unknown , or inadequate literature available Potential for significant effect on nursing infants , medication should be given with caution Not safe for nursing infants, medication is contraindicated or requires cessation of breast feeding

DENTAL QUEST Your Quest For The success In Post Graduate Dental Entrance !

DENTAL

QUEST

KCET -2011 107

119) Felypressin is a) Vasoconstrictor; Predominant effect on venous system b) Vasodilator; Predominant effect on arterial system c) Vasoconstrictor; Predominant effect on arterial system d) Vasodilator, Predominant effect on venous system Ans (a)Vasoconstrictor; Predominant effect on venous system (Malamed's Local Anesthesia5th ed page no 52) Felypressin :-a Vasoconstrictor and is more active on venous circulation than the arteriolar circulation hence minimum value for hemostasis. 120) Highest percentage of clinical failures are seen with which nerve block ? a) Buccal nerve block b) Inferior alveolar nerve block c) Mental nerve block d) Incisive nerve block Ans (b) Inferior alveolar nerve block (Malamed's Local Anesthesia5th ed page no 52) Has a highest failure rate of 15% to 20% even when properly administered inferior alveolar nerve blocks. 121) a) b) c) d) The weakest part and most vulnerable part for fracture in the mandible is Ramus of mandible Condylar neck Angle of the mandible Region of the canine tooth

K C E T 2 0 1 1

Ans (b)Condylar neck ( Text book of Oral and maxillofacial surgery by Neelima malik 1st ed page no.619) Guys, this question is bit controversial. But the word weakest and vulnerable could be the guide to find the answer. The condylar neck is vulnerable to fracture as its very slender and is actually is a safety mechanism which prevents the injury to middle cranial fossa by fracturing itself at the neck. COMMUNITY DENTISTRY 122) The tobacco preparation mainly used to clean the tooth is a) Khaini b) Mawa c) Masheri d) Dhumti Ans (c)Masheri (Essentials Of Preventive And Community Dentistry by Soben Peter 2nd ed page. 473 ) Forms of Tobacco Bidi- Most popular form of tobacco in India Chillum- A straight long conical clay pie of 10-14 cm used to smoke tobacco Chutta- Also called as Cigar where are Cheeror are smaller size cigars Cigarettes o Contains about 1gm of tobacco o Contain nicotine of 1-1.4 mg and tar around 19-27 o 31% of total tobacco grown is used to manufacture cigarettes Dhumti-- Conical cheroots and is used for reverse smoking Gudakhu--Paste of tobacco powder and used to clean the teeth mainly women of Bihar

DENTAL QUEST Your Quest For The success In Post Graduate Dental Entrance !

DENTAL
o o

QUEST

KCET -2011 108

o o o

o
o o

Hookha Water pipe or hubble-bubble Purely Indian origin product Hookli- - Clay pipe 7-10cm Kahaini Sundried tobacco and slaked lime with arecanut Smokeless tobacco form Kept in mandibular premolar vestibular area Manipuri tobacco- More incidence of oral cancer and leukoplakia Mawa - Contains shaving of arecanut chiefly Mishri/MasheriRoasted form of tobacco, and is Black in colour Used with or without catechu Primarily used for cleaning tooth Paan--most common form of smokeless tobacco in India Snuff-- Air cured and fired tobacco leaves and Applied over gingiva Zarda- Boiled tobacco leaf with lime and spice

K C E T 2 0 1 1

123) The concept which views health as 'absence from disease' is called?
a) Biomedical concept b) Ecological concept c) Psychosocial concept d) Holistic concept Ans (a)Biomedical concept (Parks textbook of Preventive and Social medicine page no 19th ed page no 12-13)

Biomedical concept- based on view Absence of disease Ecological concept- based on the fact that health is a dynamic equilibrium between man and environment. Psychological concept Based on the view that health is influenced by social, cultural, economic and political factor Holistic concept- health is composition of all the above three factors

124) The frequency of a disease or characteristic expressed per unit size of the population
or group in which it is observed is? a) Rate b) Ratio c) Proportion d) Average Ans (a)Rate ( Essentials Of Preventive And Community Dentistry by Soben Peter 2nd page .82) Rate: it is the rate frequency of a disease or characteristics expressed per unit of size of the population or group in which it is observed or it measures the occurrence of some particular event in a population during a given time period. Ratio: It expresses a relation in the size between two random quantities. Numerator is not a component of the denominator. Proportion: It is a ratio, which indicates the relation in magnitude of a part of the whole. Numerator is included in the denominator. It is usually expressed as percentage.

125) The common first approach to test causal hypothesis is?


a) b) c) Descriptive study Case-control study Cohort study

DENTAL QUEST Your Quest For The success In Post Graduate Dental Entrance !

DENTAL

QUEST

KCET -2011 109

d) Randomized controlled trial Ans (b) Case control study (Park's Textbook of Preventive and Social Medicine-17th Edition page no 61) Also called as retrospective study Both exposure and disease has occurred before start of study Study proceeds backwards from effect to cause Useful in comparison of group

126) The brushing method recommended for individuals with open interdental spaces with
missing papillae and exposed root surfaces is? a) Fones method b) Charters method c) Bass method d) Smith method Ans (b)Charters method ( Essentials Of Preventive And Community Dentistry by Soben Peter 2ND ed 452) Charters method o Open interdental spaces o Recession cases Those who are taking orthodontic treatment

K C E T 2 0 1 1

127) The cationic bisbiguanide used as a mouthwash which is effective against gram
positive and gram negative microorganisms, fungi, yeasts and viruses is? a) Chlorhexidine Gluconate b) Triclosan c) Dclmopinol d) Sanguinarine Ans (a) Chlorohexidine gluconate ( Essentials Of Preventive And Community Dentistry by Soben Peter 2nd page 461)

128) The National Cancer Registry Program (NCRP) was initiated by the Indian Council of
Medical Research in the year? a) 1972 b) 1982 c) 1992 d) 2002 Ans (b)1982 ( Shafer's textbook of oral pathology 5t ed page no.145)

129) The degree to which the health education message is perceived as trustworthy by the
receiver is called? a) Setting an example b) Comprehension c) Credibility d) Reinforcement Ans (c)Credibility (Park Textbook of Preventive and Social medicine 19th edition 715 ) Principles of Health education Credibility- It is the degree to which the message is communicated, is perceived as trustworthy by the receive Interest- Is a psychological principle that people are unlikely to listen to those things which are not to their interest Participation- Is said be the key word which is based on active learning

DENTAL QUEST Your Quest For The success In Post Graduate Dental Entrance !

DENTAL

QUEST

KCET -2011 110

Motivation- fundamental desire to learn Comprehension- its the level of understanding education and literacy of the people to whom the teaching is directed Reinforcement Leaning by doing Known to unknown- teaching simpler to more complicated things Setting an example Good human relations Feedback- a key concept of health education Community Leaders involvement 130) The Indian Government passed the "Cigarettes and other Tobacco Products Act" to prohibit advertisement of and regulate the production, supply and distribution of tobacco products in the year a) 1973 b) 1983 c) 1993 d) 2003 Ans (d) 2003 (Park's Textbook of Preventive and Social Medicine-19th Edition page no 373) 131) The world's first artificial water fluoridation plant was started at a) Culemborg b) Grand Rapids c) Muskegon d) Oak Park Ans (b) Grand rapids (Essentials Of Preventive And Community Dentistry by Soben Peter 2nd ed page 283) 1901 - Dr Fredrick Mckay discovered Colorado stains and named it as Mottled enamel 1931- Chruchil H V, Chief chemist noted the fluoride in water 1931- Trendley H Dean did shoe leather survey 1945- Worlds first artificial fluoridation plans in Grand Rapids USA

K C E T 2 0 1 1

132) The fluoride concentration in 8% Stannous Fluoride, topical application, is


a) 1000 ppm b) 9200 ppm c) 19500 ppm d) 12300 ppm Ans (c)19500 ppm ( Essentials Of Preventive And Community Dentistry by Soben Peter 2nd ed page.315) Topical Fluoride agents

Property
Fluoride percentage Ppm Application time

APF
1.23% 12,300 1 or year 2 2%

Na F
9200 At age of 3,7,11,13 week( once in 4 weeks in each age) 29%

Sn F2
8% 19,500 1 or 2 year

Caries reduction potential

28%

30%

DENTAL QUEST Your Quest For The success In Post Graduate Dental Entrance !

DENTAL

QUEST

KCET -2011 111

133) Milk Fluoridation was introduced by a) Knutson b) Bibby c) Wespi d) Zeigler Ans (d)Zeigler (Fluoride in dentistry. Fejer- skov 2nd ed page no. 305)

134) The WHO periodontal examination probe weighs?


a) 5 gms b) 10 gms c) 15 gms d) 20 gms Ans (a) 5 gms (Essentials Of Preventive And Community Dentistry by Soben Peter 2nd page.174) CPITN probe or WHO periodontal examination probe First described by WHO (TRS -621 1978). Purposes, o Measurement of pocket depth o Detection of subgingival calculus. Light weight (5 gms). Designed for gentle manipulation of the often v sensitive soft tissues of teeth. o Colour coding with a Black band starting at 3.5 mm and ends at 5.5 mm. o Has a ball tip of 0.5 mm diameter for detection of subgingival calculus. o Variant- two extra marking at 8.5 mm and 11.5 mm from the working tip for use of a detailed assessment and recording of deep pockets. The Joint Working Committee of WHO/ FDI have named two types of WHO probeCPITNE - Epidemiological probe with 3.5 mm and 5.5 mm markings, CPITNC - Clinical probe with additional 8.5 mm and 11.5 mm marking

K C E T 2 0 1 1

135) The ratio between the achievement of program activity and the desired level which, the
planners had proposed would result from the program is called? a) Effectiveness b) Efficiency c) Appropriateness d) Adequacy Ans (a)Effectiveness ( Essentials Of Preventive And Community Dentistry by Soben Peter 2nd page 652

136) The dye used in the Salivary Reductase test is?


a) Bromocresol green b) Diazo resorcinol c) Methylene blue d) Basic fuschin Ans (b) Diazo resorcinol (Essentials Of Preventive And Community Dentistry by Soben Peter 2nd page. 377) o Bromocresol green- Calorimetric Snyder test o Diazo resorcinol- Salivary Reductase test o Salivary Reductase test o A Susceptibly test and measures the activity of reducatse enzyme o A kit is available under the trade name Treatex

Color
Blue Orchid

Time
15 min. 15 min.

Score
1 2

Caries activity
Non conducive Slightly conducive

DENTAL QUEST Your Quest For The success In Post Graduate Dental Entrance !

DENTAL
Red Red Pink or White

QUEST
15 min. Immediately Immediately 3 4 5

KCET -2011 112


Moderately conducive Highly conducive Extremely conducive

137) Which topical fluoride agent makes use of thixotrophic properties for better 'penetration into inter proximal areas? a) Sodium fluoride solution b) Fluoride foam c) Acidulated phosphate fluoride gel d) Fluoride varnish Ans (c) Acidulated phosphate fluoride gel ( Essentials Of Preventive And Community Dentistry by Soben Peter 2nd page 303) Thixotropic property is the ability of the material to show decreased viscosity and increased flow under stress. APF is available in various forms and also available has thixotropic property.

K C E T 2 0 1 1

PROSTHODONTICS
138) For strength and rigidity, the vertical height of a finished lingual bar should be at least?
a) 6mm b) 7mm c) 4mm d) 5mm Ans (c) 4mm (Stewart's Clinical Removable Partial Prosthodontics page no. 33 )

139) A reliable guide for positioning maxillary anterior teeth in complete dentures is ?
a) b) c) Rugae Incisive papilla Maxillary ridge Fovea palatinae

d)

Ans (b) Incisive papilla ( Treatment for Edentulous Patients - Zarb and bolender12th ed page no 309) Incisive papilla In the natural dentition the labial surface of the upper central incisor is about 10 - 12 mm anterior to the center of the incisive papilla After extraction of the teeth and the subsequent bony remodeling. The labial surface of incisors should be placed 8 - 10 mm anterior to it. The incisive papilla is one of the most useful biometric guides

140) Average masticatory load for natural teeth is around?


a) 10 kg b) 20 kg c) 30 kg d) 40 kg Ans (b)20 kg ( Treatment for Edentulous Patients - Zarb and bolender 12th ed page no 10) o Average masticatory load for natural teeth 20 kg or 44 lb o Average masticatory load by complete denture 6-8 kg or 13to 16 lb o 5-6 times less force in denture

141) Articulated diagnostic casts help in the following EXCEPT?


a) b) c) Diagnostic wax up to evaluate outcome Detailed analysis of Occlusal plane and occlusion Fabrication of wax patterns for casting

DENTAL QUEST Your Quest For The success In Post Graduate Dental Entrance !

DENTAL

QUEST

KCET -2011 113

d) Performing mock tooth preparation Ans (c)Fabrication of wax patterns for casting ( Textbook of Complete dentures Zarb Bolender 12th ed page no 83) Obviously a diagnostic cast cannot be used for fabrication of wax pattern as it is made of dental plaster type II and does not have sufficient abrasion resistance or strength for making wax pattern for it. And as the name its just used for diagnosis and treatment planning but not in later stages.

142) Least hygienic Pontic ?


a) Sanitary b) Bullet c) Spheroidal d) Saddle Ans (d)Saddle (Fundamentals of Fixed Prosthodontics by Shillingburg page no 488-491)

K C E T 2 0 1 1

143) Advantage of a palatal strap?


a) Increased rigidity with less bulk b) Reduction of bulk c) Better simulation of the underlying tissues d) Easy manipulation Ans (a) Increased rigidity with less bulk (Stewart's Clinical Removable Partial Prosthodontics page no. 26-27 ) PALATAL STRAP Most versatile maxillary major connector Should not be less than 8mm Preferred in unilateral distal extension cases ( and not bilateral ) Advantages o Provided greatest resistance with less bulk which is based on L principle o Little indirect retention is provided. Disadvantage Papillary hyperplasia

144) Relief under a maxillary Major Connector?


a) Is generally not required b) Is required c) May be used d) May not be used Ans (a) Is generally not required (Stewart's Clinical Removable Partial Prosthodontics page no. 32) Relief in not always required for maxillary major connectors but more common for mandibular major connectors.

145) Major connector is a poor choice for distal extension RPD?


a) Single palatal strap b) Complete Palatal coverage c) Horse shoe shaped or U shaped MC d) Anterior Posterior or Double Palatal bar Ans (c) Horse shoe shaped or U shaped MC (Stewart's Clinical Removable Partial Prosthodontics page no. 29) Horse or U shaped Major connector Borders always placed in valley of rugae. Advantages o Derives some vertical support from tissue

DENTAL QUEST Your Quest For The success In Post Graduate Dental Entrance !

DENTAL

QUEST

KCET -2011 114

o Indirect retention present Disadvantages o It has a tendency to spread or straighten hence poor choice in distal extension cases o Less resistant to flexing 146) Recommended convergence between opposing axial walls which optimize retention is a) 6 degree b) 30 degree c) 15 degree d) 45 degree Ans (a) 6 degree (Fundamentals of Fixed Prosthodontics by Shillingburg page no 119) The axial walls of preparation for cast metal or ceramic restoration should allow placement by having an angle of convergence of 3o per wall. Hence the two opposing walls will give a taper of 6o taper together.

K C E T 2 0 1 1

147) In case of a hard median suture line and an inoperable torus the major connector of
choice is? a) Single palatal strap b) Complete Palatal coverage c) Horse shoe shaped or U shaped MC d) Anterior Posterior or Double Palatal bar Ans (c) Horse shoe shaped or U shaped MC (Stewart's Clinical Removable Partial Prosthodontics page no. 28 ) Horse shaped major connector can be used in hard median suture line without losing vertical support of strength.

148) Height of contour is a term coined by?


a) Kennedy b) Cummer c) Devan d) Applegate Ans (a) Kennedy (Stewart's Clinical Removable Partial Prosthodontics page no. 56 ) Height of contour is the greatest bulge or diameter of crown when viewed from specific angle.

149) The functional Occlusal force applied to the periodontal tissues during one deglutition
is for duration of seconds? a) 0.3 b) 0.5 c) 1.0 d) 1.5 Ans (c) 1.0 (Treatment for Edentulous Patients - zarb and bolender 12th ed page no 10) 150) The first person to employ mechanical device to determine the relative parallelism of tooth surface was a) Fortunati b) Ney c) Applegate d) Kratochvil Ans (a) Fortunatti (Stewart's Clinical Removable Partial Prosthodontics page no. 219) Fortunati used parallelometer in 1918 to demonstrate correct method for charting clasp placement.

DENTAL QUEST Your Quest For The success In Post Graduate Dental Entrance !

DENTAL

QUEST

KCET -2011 115

Commercially Ney instrument was marketed in 1923 which is most commonly used followed by Wills surveyor by Jelenko. 151) Occlusal offset is an anterior partial veneer crown may generally be necessary to provide a) Retention b) Conservation of tooth structure c) Esthetic d) Structural durability Ans (d) Structural durability (Fundamentals of Fixed Prosthodontics by Shillingburg page no 159) In preparation of Maxillary Posterior Three quarter crowns, the Occlusal offset of 1 mm wide on lingual inclines of facial surface with 171 L bur which provides reinforcement for metal. 152) Facial Occlusal line in the maxillary arch is formed by a) Stamp cusps b) Centric holding cusps c) Supporting cusps d) Non-supporting cusps Ans (d) Non-supportive cusps ( Sturdevants art and science of operative dentistry 4th ed page no. 38) Supporting cusps (centric, holding, or stamp cusps) - Cusps that contact the opposing teeth along the central fossa occlusal line. Nonsupporting cusps (noncentric or nonholding cusps)- cusps that overlap the opposing teeth For example, the mandibular facial occlusal line identifies the mandibular supporting cusps, whereas the maxillary facial cusps are non-supporting cusps. The names are usually applied only to posterior teeth to distinguish the difference in function between the two rows of cusps.

K C E T 2 0 1 1

PERIODONTOLOGY
153) An example of vertical method of tooth brushing?
a) Leonard's method b) Hone's method c) Smith's method d) Bass method Ans (a) Leonards method (Essentials of Preventive and Community Dentistry by Soben Peter 2nd page 451) Tooth brushing techniques were classified by GREEN J.C in 1966 as :The Roll technique: Modified stillman/rolling stroke The vibratory technique: Stillman,charters & bass method The circular technique : Fones method The vertical technique: Leonards method The horizontal technique: Scrub-brush method The physiological technique: Smiths method

DENTAL QUEST Your Quest For The success In Post Graduate Dental Entrance !

DENTAL

QUEST

KCET -2011 116

154) All of the following organisms can be detected by BANA analysis EXCEPT? a) Tannerella.forythia b) Porphyromonas.gingivalis c) Capnocytophaga d) Actinomycetemcomitans Ans (d)Actinomycetemcomitans (Carranzas clinical periodontology 10th ed page no 591) Tannerella forythia, Porphyromonas gingivalis, Capnocytophaga and treponema denticola species share common enzymatic profile i.e. all have TRYPSINLIKE enzyme. The activity of this enzyme can be measured with hydrolysis of the colourless substrate N-benzoyl-d L-arginine-2-napthylamide (BANA) which on hydrolysis releases the chromophore beta-naphthylamide,which turns orange red when a drop of fast garnet is added to the solution. LOESCHE proposed the use of this test as a marker of disease activity. He showed shallow pockets exhibited only 10% positive BANA reactions ,whereas deep pockets (7mm) exhibited 80% to 90% positive BANA reactions. BECK used BANA test as a risk indicator for periodontal attachment loss.

K C E T 2 0 1 1

155) Ideal thickness of a free gingival graft obtained from the palate is?
a) 0.5-1 mm b) 1-1.5 mm c) 1.5-2 mm d) 2.0-2.5 mm Ans (b) 1-1.5 mm ( Carranzas clinical periodontology 9th ed page no 854-855)

Palate is the common site from donor tissue. Proper thickness is important for survival of the graft. The ideal thickness of a graft is between 1.0 and 1.5 mm

156) Fusion of cementum and alveolar bone with the obliteration of periodontal ligament is
called? a) Avulsion b) Extrusion c) Ankylosis d) Luxation Ans (c) Ankylosis (Carranzas clinical periodontology 9thth ed page no 44) Ankylosis :- fusion of the cementum and alveolar bone with obliteration of the periodontal ligament is termed ankylosis. Occurs in teeth with cemental resorption which suggests that it may represent a form of abnormal repair. May also develop after chronic periapical inflammation, tooth replantation and occlusal trauma and around embedded teeth. Results in resorption of the root and its gradual replacement by bony tissue, because of this reimplanted teeth that ankylosis will lose their roots after 4 to 5 years and exfoliate.

157) Flaps that accomplish the double objectives of pocket elimination and increasing the
width of attached gingiva is? a) Apically displaced flap b) Coronally displaced flap

DENTAL QUEST Your Quest For The success In Post Graduate Dental Entrance !

DENTAL

QUEST

KCET -2011 117

c) Papilla preservation 11 ap d) Undisplaced Hap Ans (a)Apically displaced flap (Carranzas clinical periodontology 1oth ed page no 926-927) Periodontal flaps can be classified as follows Bone exposure after flap reflection:1. Full thickness flaps:-indicated when respective osseous surgery is contemplated 2. Partial thickness/split thickness flaps :-indicated when the flap is to be positioned apically or when the operator does not want to expose the bone. Placement of flap after surgery:1. Non displaced flaps: - when the flap is returned and sutured in its original position. 2. Displaced flap:-placed apically, coronally or laterally to their original position. Apically displaced flaps preserve the outer portion of the pocket wall and transforming it into attached gingiva. These flaps accomplish the double objective of pocket elimination and increasing the width of attached gingiva. Based 1. a. b. c. d. 2. on management of the papilla:Conventional flap :- includes Modified Widman , Undisplaced flap, Apically displaced flap Flap for reconstructive procedures. Papilla preservation flap

K C E T 2 0 1 1

158) The sixth complication of diabetes mellitus is a) Retinopathy b) Renal failure c) Ischemic heart diseases d) Periodontitis Ans (d)Periodontitis (Carranzas clinical periodontology 10th ed 285)

Uncontrolled diabetes includes reduction in defense mechanisms and increased susceptibility to infections. Periodontal disease is considered to be the sixth complication of diabetes. Children with type 1 diabetes have more destruction around incisors and 1st molars ACCUMULATED GLYCATION END PRODUCTS (AGEs) that plays a central role in classic complication of diabetes. Remember no alteration in IgA, IgG or IgM has been found in diabetic patients.

159) A drug that exerts anticollagenase activity?


a) b) c) d) Penicillin Metronidazole Erythromycin Tetracycline

Ans (d)Tetracycline (Carranzas clinical periodontology 10th ed pg no 803) Tetracycline have the ability to concentrate in the periodontal tissues and inhibit the growth of AAC. And has anticollagenase effect which inhibits tissue destruction and may aid in bone regeneration. Some key points on Tetracycline: Bacteriostatic Concentration in gingival cervices is 2 to 10 times than is serum. Used as an adjunct in treatment of LAP

DENTAL QUEST Your Quest For The success In Post Graduate Dental Entrance !

DENTAL

QUEST

KCET -2011 118

Systemic tetracycline can eliminate tissue bacteria and arrest bone loss and suppress AAC levels in conjugation with scaling and root planning. Requires 250 mg four times daily (qid). 160) The function of Langerhan's cells is? a) Melanin synthesis b) Protein synthesis c) Antigen presentation to neutrophils d) Antigen presentation to lymphocytes Ans (d)Antigen presentation to lymphocytes (Carranzas clinical periodontology 1oth ed page no 52) Langerhans cells are dendritic cells located among keratinocytes at all suprabasal levels and belong to mononuclear phagocyte system (RES) as modified monocytes derived from the bone marrow. Important role in immune reaction as antigen presenting cells for lymphocytes. Contain g-specific granules (birbecks granules) Have marked adenosine triphosphatase activity. Remember:-They are absent from junctional epithelium of normal gingiva.

K C E T 2 0 1 1

161) A replacement drug used as a substitute for cyclosporine exhibiting less severe
gingival enlargement is? a) Methotrexate b) Tacrolimus c) Nifedipine d) Bleomycin Ans (b) Tacrolimus (Carranzas clinical periodontology 10th ed page no 378) Cyclosporine causes gingival enlargement and also causes major side effects like nephrotoxicity, hypertension and hypertrichosis. Tacrolimus is used effectively which is although nephrotoxic but causes less gingival overgrowth, hypertrichosis and severe hypertension. Some key points on Cyclosporine:1. Administered I.V. or orally and dosages greater than 500 mg/day have been reported to cause gingival overgrowth 2. It causes more vascularized gingival overgrowth than phenytoin. 3. Enlargement occurrence 25-70% 4. Affects children more 5. Enlargement is related more to its plasma concentration and enlargement is indicated to be a hypersensitivity response to cyclosporine.

162) Stage IV of ANUG according to Horning and Cohen is characterized by?


a) Necrosis of the tip of interdental papilla b) Necrosis extending to the attached gingiva c) Necrosis exposing alveolar bone d) Necrosis perforation skin of cheek Ans (b) Necrosis extending to the attached gingiva (Carranzas clinical periodontology 10th ed page no 392) Horning & Cohen extended the staging of these oral necrotizing diseases as follows:o Stage 1: Necrosis of the tip of interdental papilla. o Stage 2: Necrosis of entire papilla. o Stage 3: Necrosis extending to gingival margin. o Stage 4: Necrosis extending also to attach gingiva. o Stage 5: necrosis extending into buccal or labial mucosa.

DENTAL QUEST Your Quest For The success In Post Graduate Dental Entrance !

DENTAL
o o

QUEST

KCET -2011 119

Stage 6: Necrosis exposing alveolar bone. Stage 7: Necrosis perforating skin of cheek.

According to Horning and Cohen:Stage 1 is NUG Stage 2 may be either NUG OR NUP Stage 3 and 4 corresponds to NUP Stage 5 and 6 corresponds to necrotizing stomatitis and Stage 7 would corresponds to NOMA 163) The lactoperoxidase thiocynate system present in saliva is against a) A.actinomycetemcomitans b) P.gingivalis c) Streptococcus d) Actinomyces Ans (b)P.gingivalis (Carranzas clinical periodontology 10th page no ed 349) Lactoperoxidase thiocynate system is bactericidal to some strains of lactobacillus and streptococcus by preventing accumulation of lysine and glutamic acid both of which are must for bacterial growth. 164) In a biofilm the capacity of bacteria to communicate with each other is called a) Quorum sensing b) Conjugation c) Transmission d) Transformation Ans (a) Quorom sensing (Carranzas clinical periodontology 1oth ed page no 150-151) In a biofilm, bacteria communicate with each other by quorum sensing which involves regulation of expression of specific genes through the accumulation of signaling compounds that mediate intercellular communications which when reaches a threshold level (quorum cell density), gene expression is activated. 165) Which of the following acute infections is contagious? a) Acute gingival abscess b) Necrotizing ulcerative gingivitis c) Primary herpetic gingivostomatitis d) Acute pericoronitis Ans (c) Primary herpetic gingivostomatitis (Carranzas clinical periodontology 10th ed page no 400) Primary herpetic gingivostomatitis is contagious. 166) Prognosis is a) Most likely cause of the disease b) Least likely cause of the disease c) Estimation of the likely course of the disease d) None of the above Ans (c) Estimation of the likely course of the disease (Carranzas Clinical Periodontology 10th ed page no. 114, 614)

K C E T 2 0 1 1

167) A direct connection between living bone and load-bearing endosseous implant at the
light microscopic level? a) Osteosynthesis b) Osteogenesis

DENTAL QUEST Your Quest For The success In Post Graduate Dental Entrance !

DENTAL

QUEST

KCET -2011 120

c) Osseointegration d) Osseoinduction Ans (c) Osseointegration (Carranzas clinical periodontology 9th ed page no 885) The relationship between endosseous implants and bone consists of two mechanisms: OSSEOINTEGRATION:- when the bone is in intimate but not ultrastructural contact with the implant..Osseointegration refers to the direct contact of bone and implant at the light microscopic level. FIBROSSEOUS INTEGRATION :- soft tissues such as fibres and/or cells,are interposed between two surfaces. OSSEOINTEGRATION CONCEPT was proposed by BRANEMARK and called FUNCTIONAL ANKYLOSIS by SCHROEDER which states that there is an absence of connective tissue or any nonbone tissue in the interface between the implant and the bone. Note- osseointegration never occurs 100% of the implant surface. successful cases have been between 30 and 95% of the implant surface, as measured by light microscopy, in contact with the bone.

K C E T 2 0 1 1

RADIOLOGY
168) a) b) c) d)
Following exposure, the X-rays interact with patient tissues within? 10-13 seconds 1010 seconds 103 seconds 100 seconds Ans (a)10-13 seconds (Oral Radiology- Principles and Interpretation White & Pharoah page no 25) The initial interaction between ionizing radiation and matter occurs at the level of the electron within the first 10-13 second after exposure. Biologic effects of ionizing radiation may be divided into two broad categories: deterministic effects and stochastic effects. Deterministic effects are those effects in which the severity of response is proportional to the dose. Stochastic affects the probability of occurrence of a change, rather than its severity is dose dependent i.e. a person will have or does not have a condition (all-or-none) 169) The traditional unit of equivalent dose is a) Rem b) Rad c) Roentgen d) Sievert Ans (a)Rem (Oral Radiology- Principles and Interpretation White &Pharoah page no 20) Quantity SI unit Traditional unit Exposure Air kerma (Gy) Roentgen (R) Absorbed dose Equivalent dose Effective dose Radioactivity CONVERSION 1 Gy = 100 rad Gray (Gy) Sievert (Sv) Sievert (Sv) Becquerel (Bq) Curie (Ci) Rad Rem

DENTAL QUEST Your Quest For The success In Post Graduate Dental Entrance !

DENTAL

QUEST

KCET -2011 121

1 rad = 0.01 Gy (1 cGy) 1 Gy - I 00 rad 1 rad e 0.0 1Gy ( 1cGy) 1 Sv= 100rem 1 rem = 0.01 Sv (1 cSv) 1 Bq = 2.7 X 10-11Ci Ci = 3.7 x 101DBq The SI unit of exposure is Roentgen Air kerma Curie Sievert

170) a) b) c) d)

K C E T 2 0 1 1

Ans (b)Air kerma (Oral Radiology- Principles and Interpretation White & Pharoah page no 20 See above explanation

171) Penny test is the quality assurance test to detect?


a) Fixer depletion b) Unsafe illumination c) Machine malfunction d) Contaminated solution Ans (b)Unsafe illumination (Oral Radiology- Principles and Interpretation White & Pharoah page no 112) Penny test is done to Check improper Darkroom Safelighting conditions. Done once in a month and is a simple test to prevent fogging of films which can be due to improper safelight filter, excessive exposure to safelight or stray light from other source in darkrooms. Steps Keep an exposed film just taken out of its packet in the area where the films are usually unwrapped Place a penny on the film and leave it in this position usually about 5 minutes. After developing if the image of the penny is visible on the film, the room does not have proper safelight.

172) Identification of gross osseous changes from a lateral aspect can be viewed by which
projection? a) Transcranial b) Transorbital c) Transpharyngeal d) Translaryngeal Ans (a)Transcranial (Oral Radiology- Principles and Interpretation White & Pharoah page no 543) Transcranial Projection A sagittal view of the lateral aspects of the condyle and temporal components can be obtained. X-ray beam is directed downward from the opposite side, through the cranium and above the petrous ridge of the temporal bone, at a 25-degree positive angle centered through the joint. The horizontal direction of the beam may be individually corrected for the condylar long axis an average 20-degree anterior angle may be used. It includes projections of both TMJs in the closed and maximally open positions routinely. The central and medial aspects of the joint are projected inferiorly, and only lateral joint contours are visible due to positive angulation.

DENTAL QUEST Your Quest For The success In Post Graduate Dental Entrance !

DENTAL

QUEST

KCET -2011 122

Useful for identifying gross osseous changes on the lateral aspect of the joint only, displaced condylar fractures, and range of motion. Panoramic Projection Provides an overall view of the teeth and jaws, hence a screening projection No information about condylar position or function is provided Panoramic view does not provide an adequate because of superimposition by zygomatic arch and skull base Transpharyngeal (parma) Projection It provides sagittal view of the medial pole of the condyle. The beam is directed superiorly at -5 degrees through the sigmoid notch of the opposite side and 7 to 8 degrees from the anterior with maximal opening of mouth The medial aspect of the condyle can be viewed because of negative beam angulation. Temporal component is not imaged well Effective for visualizing erosive changes of the condyle Transorbital Projection The patient's head is tilted downward 10 degrees so that the canthomeatal line is horizontal. Beam is directed from the front of the patient through the ipsilateral orbit and TMJ of interest. The film cassette is placed behind the patient's head, perpendicular to the x-ray beam. The patient opens maximally and protrudes the mandible, thereby avoiding superimposition of the articular eminence or skull base on the condyle Very good for visualizing condylar neck fractures as entire mediolateral dimension of the articular eminence, condylar head, and condylar neck is visible. Adjuvant to diagnose degenerative changes or other anomalies of TMJ. A similar projection is the reverse open Towne's projection, which sometimes is used to image condylar neck fractures, mainly if medial displacement is suspected Submentovertex (Basal) Projection It gives a view of the skull base and condyles superimposed on the condylar necks and mandibular rami. Used to determine the angulations of the long axis of the condylar head for corrected tomography This is an adjuvant to find the TMJ changes in facial asymmetries, condylar displacement, or rotation of the mandible in the horizontal plane associated with trauma or orthognathic surgery 173) Which intraoral radiographic technique follows the "Rule of Isometry"? a) Paralleling technique b) Occlusal radiographs c) Bisecting angle technique d) Cephalometric radiographs Ans (c) Bisecting angle technique ( oral radiology by white and pharoh 5th ed page no.125) Cieszynski's rule of isometry :

K C E T 2 0 1 1

Used in bisecting-angle technique of radiograph. Principle- two triangles are equal when they share one complete side and have two equal

angles. An imaginary line that bisects the angle made by the film and the long axis of the teeth is constructed and central ray of the beam is directed at right angles to this bisector which forms two triangles with two equal angles and a common side (the imaginary bisector). Disadvantage o To correctly reproduce the length of each root of a multi rooted tooth, the central beam must be angled differently for each root. o The alveolar ridge often projects more coronally than its true position

DENTAL QUEST Your Quest For The success In Post Graduate Dental Entrance !

DENTAL

QUEST

KCET -2011 123

CONSERVATIVE AND ENDODONTICS


174) a) b) c) d)
Cardiac Pain may be transmitted to the jaw due to the overlapping of? 5th cranial nerve, third cervical nerve and first thoracic nerve 7th cranial nerve, third cervical nerve and first thoracic nerve 5th cranial nerve, second cervical nerve and first thoracic nerve 7lh cranial nerve, second cervical nerve and first thoracic nerve

Ans (a) 5th cranial nerve, third cervical nerve and first thoracic nerve (INGLES ENDODONTICS 5TH ed pg no 316 and fig 8-19) Cardiac pain can be referred as far away as the jaws. Potential convergence of nerve fibers from different sensory dermatomes in the dorsal horn of the spinal cord provides a basis for a "convergence-projection" phenomenon for the pain referral. The thoracic dermatomes, innervating the chest and arms, overlap with the cervical dermatomes, some of which innervate the arm and shoulder, as well as part of the lower face. The second cervical dermatome, in turn, slightly overlaps with the fifth cranial nerve that innervates the entire oral complex

K C E T 2 0 1 1

175) One of the following statements regarding HIV is NOT true.


a) When used properly, all disinfectants except some quaternary ammonium compounds are said to inactivate HIV in less than 2 minutes b) HIV is killed by all methods of sterilization c) Aerosols produced during dental treatments have been found to transmit HIV infection d) HIV has been transmitted by blood contaminated fluids that have been heavily spattered or splashed Ans (c) Aerosols produced during dental treatments have been found to transmit HIV infection (Sturdevants art and science of operative dentistry 4th ed page no. 358) HIV RISKS FOR DENTAL PATIENTS The risk of transmission in dentistry is extremely low. Reasons 1. HIV usually has been found in very low levels in blood of infected persons than the HB virus 2. In dried infected blood, 99% of HIV gets inactive in approximately 90 minutes but in wet environment it may survive for more than 2 days like in needles. 3. HIV is killed by all methods of sterilization except some quaternary ammonium compound it gets inactivate in less than 2 minutes. 4. HIV has been transmitted by blood-contaminated fluids that have been heavily spattered or splashed. 5. Aerosols, produced during dental treatments, have not been found to transmit FIB or HIV infection. 6. Barriers have proven successful in protecting dental personnel against HIV Recently for the immunocompromised person, as well as for dental personnel airborne transmission of multidrug-resistant Mycobacterium tuberculosis has been a matter of concern 176) Whenever the caries cone in enamel is larger or at least the same size as that in dentin, it is called as a) Residual caries b) Recurrent caries c) Forward caries

DENTAL QUEST Your Quest For The success In Post Graduate Dental Entrance !

DENTAL

QUEST

KCET -2011 124

d) Backward caries Ans (c) Forward caries ( Sturdevants art and science of operative dentistry 4th ed page no. 274) Backward Caries. When the spread of caries along the DEJ exceeds the caries in the contiguous enamel, caries extends into this enamel from the junction. Forward Caries. Forward caries is where the caries cone in enamel is larger or atleast the same size as that in dentin. 177) "Right rear' operator position refers to a) 1 O'clock position b) 5 O'clock position c) 7 O'clock position d) 11 O'clock position Ans (d) 11 O clock position (Sturdevants art and science of operative dentistry 4th ed page no. 433) Right Front Position. (7o clock) o For mandibular anterior teeth mandibular posterior teeth (especially on the right side), and maxillary anterior teeth. Right Position. (9oclock) o For operating on the facial surfaces of the maxillary and mandibular right posterior teeth and the occlusal surfaces of the mandibular right posterior teeth. Right Read Position. (11oclock) o Position of choice for most operations. o The lingual and incisal (occlusal) surfaces of the maxillary teeth observed in mirror. o Direct vision for particularly on the left side, Direct Rear Position (12o Clock) For lingual surfaces of mandibular anterior teeth. 178) The disadvantage of winged rubber dam retainer is that it interferes with the placement of a) Matrix bands b) Dental floss c) Rubber dam frame d) Rubber dam sheet Ans (a) Matrix bands (Sturdevants art and science of operative dentistry 4th ed page no. 447) A disadvantage of the winged retainer is it interfere with the placement of o Matrix bands o Band retainers o And wedges Advantage- extra retraction of rubber dam 179) Discolored areas seen through the enamel in the teeth that have amalgam restorations are known as a) Amalgam tattoos b) Amalgam overhangs c) Amalgam blues d) Amalgam ditchings

K C E T 2 0 1 1

the mouth

DENTAL QUEST Your Quest For The success In Post Graduate Dental Entrance !

DENTAL

QUEST

KCET -2011 125

Ans (c) Amalgam blues (Sturdevants art and science of operative dentistry 4th ed page no. 409) Amalgam blues - Teeth having amalgam restorations may show discolored areas seen through the enamel and is termed as Amalgam Blues which is due to leaching of corrosion products of amalgam into the dentinal tubules. It can also occur when the enamel has no dentin support in areas like undermined cusps,marginal ridges and regions adjacent to proximal margins where the colour is seen through the thin enamel. They are not considered as caries. . Amalgam Tattoo - Implantation of dental amalgam in soft tissue adjacent to the teeth and termed as also termed as Localized argyrosis 180) Which of the following item in the dental operatory cannot be classified as a semicritical item ? a) Suction lip b) Hand piece c) Endodontic file d) Water syringe tip Ans (c) Endodontic file (INFECTION CONTROL by Caris MILLER 3rd ED PAGE NO.195) and (Sturdevants art and science of operative dentistry 4th ed page no. 366) Critical InstrumentsThese are the instruments which contact or cut tissues or penetrate tissues are considered to be critical items and always require thorough cleaning and sterilization for reuse. Eg- Endodontic file, Scalpel Semi-Critical InstrumentsThese are the item which touch mucosa but does not cut or penetrate the tissue. Sterilization should be considered but if instrument are sensitive then high level disinfection should be carried. Eg- air/water syringe tip, suction tips, handpiece Non- Critical InstrumentsThese are the environmental surfaces such as chairs, benches, floors, walls, and supporting equipment of the dental unit that are not ordinarily touched during treatments. Surface disinfection is adequate. Eg- X ray Stand, Light source

K C E T 2 0 1 1

181) One of the following statement regarding pin retained restoration is NOT true?
a) Pinholes should never be prepared at different levels on the tooth b) A minimum of 0.5 mm clearance should be provided around the circumference of the pin for adequate condensation of the amalgam c) Pinhole should located on a flat surface that is perpendicular to the proposed direction of the pinhole d) The pinhole should be positioned no closer than 1 mm to DEJ and no closer than 1.5 mm to the external surface Ans (a) Pinholes should never be prepared at different levels on the tooth (Sturdevants art and science of operative dentistry 4thth ed page no. 774) o Occlusal clearance of 2 mm of amalgam over the pin o Ideally, pinholes should be located halfway between the pulp and the DEJ or external surface of the root but at least 1 mm of sound dentin around the circumference of the pinhole. o The pinhole should be positioned no closer than 0.5 to 1 mm to the DEJ or no closer than 1 to 1.5 mm to the external surface of the tooth, whichever distance is greater. o The pinhole should be parallel to the adjacent external surface of the tooth. o 0.5 mm clearance around the circumference of the pin for adequate condensation of amalgam should be given

DENTAL QUEST Your Quest For The success In Post Graduate Dental Entrance !

DENTAL

QUEST

KCET -2011 126

o Pinholes should be prepared on a flat surface that is perpendicular to the proposed direction of the pinhole. o When three or more pinholes are placed, they should be located at different vertical levels on the tooth as it helps to reduce the stresses o The optimal interpin distance is 3 mm for the Minikin (0.019 inch [0.48 mm]) pin and 5 mm for the Minim (0.024 inch [0.61 mm]) pin o External perforation may result from pinhole placement Mesial concavity of the maxillary first premolar Midlingual and midfacial bifurcations of the mandibular first and second molars Midfacial, midmesial, and mid-distal furcations of the maxillary first and second molars. Pulpal penetration may result from pin placement at the mesiofacial corner of the maxillary first molar and the mandibular first molar. 182) All of the following tests are used to determine the status of the pulp except? a) Heat test b) Electric test c) Cold test d) Anesthetic test Ans (d) Anesthetic test (Grossmans Endodontic Practice 12th ed page no.70) Anesthetic test is done when usual test have failed to identify the tooth having pain but it is not used not specifically to identify the status of the pulp. One tooth at a time is anesthetized until the pain disappears and is localized to a specific tooth .Intraligamentry or infiltration is used, the posterior last tooth is anesthetized first, and if pain does not disappear then continued to the mesial tooth till the specific tooth is identified. It is also used to differentiate tooth from upper or lower arch, in which an mandibular block is given first. It is noninvasive in contrast to test cavity 183) A Peeso reamer is used for a) Preparation of post space in the root canal for receiving a post b) Enlarging the canal orifices during access cavity preparation c) Removal of the lingual shoulder to improve the access in anterior teeth d) Cleaning and shaping of the cervical third of the root canal Ans (a)Preparation of post space in the root canal for receiving a post (Grossmans Endodontic Practice 12th ed page no.232) Largo drills or Peeso Reamers Steel instruments for the contra angled handpiece Blades are spread over a wider surface than the Gates-Glidden drills Has a non-cutting tip Available from #1 , # 6 (0.2 mm increase per number) # 1 - 0.7 mm , #2 - 0.9 mm, # 3 - 1.1 mm, # 4 1.3 mm, # 5 1.5 mm, # 5 1.7 mm Easily cause root stripping if not used carefully. Largo drills / Peeso Reamers are mainly used for preparation of the dowel space (post space) to remove gutta-percha from the canal. 184) In chronic hyperplastic pulpitis, the response to vitality tests will be a) Normal response b) Hyper response c) Reduced response d) Normal to hyper response Ans (c)Reduced response (Grossmans Endodontic Practice 12th ed page no. 89 90)

K C E T 2 0 1 1

DENTAL QUEST Your Quest For The success In Post Graduate Dental Entrance !

DENTAL
185) a) b)

QUEST

KCET -2011 127

The advantage of silver point as obturating material is that it Can be used in fine tortuous canals Is easier to retrieve if retreatment becomes necessary c) Gives a perfect seal at the apical l/3rJ of the root canal d) Provides better adaptation to canal walls compared to Gutta Percha Ans (a) Can be used in fine torturous canals (Grossmans Endodontic Practice 11th ed page no.251) 1933 Elmer A. Jasper fabricated the Silver point Technique Advantage Stiffer than gutta-percha Easier to insert in very narrow/ fine tortuous canals Disadvantages Poor lateral seal Corrodes Higher failure rates Difficulty in retrieving cones in case of retreatment 186) If a tooth has abnormal mobility, but is not displaced from the socket it is termed as a) Concussion b) Subluxation c) Extrusive luxation d) Partial avulsion Ans (b)Subluxation (Grossmans Endodontic Practice 12th ed page no.375) Concussion - Implies no displacement, normal mobility, sensitivity to percussion. Subluxation- Implies sensitivity to percussion, increased mobility, no displacement. Lateral luxation - Implies displacement labially, lingually, distally or incisally. Extrusive luxation- Implies displacement in a coronal direction. Intrusive luxation - Implies displacement apically into the alveolus Avulsion or extra articulation- The complete displacement of tooth out of its socket 187) The type of root resorption seen after bleaching is a) Cervical root resorption b) Lateral root resorption c) Apical root resorption d) Internal root resorption Ans (a) Cervical root resorption (Ingle's Endodontics 6th ed page no 1391) 188) In an endodontic file, D, will be a)At the tip of the instrument b)At 2mm from the tip of the instrument c) At 3mm from the tip of the instrument d)At 16 mm from the tip of the instrument Ans (d) At 16 mm from the tip of the instrument (Ingle's Endodontics 6th ed page no 813) o Instruments numbering system for endodontic instrument was revised last in 2002 using numbers from 6 to 140, is based on the diameter of the instruments in hundredths of a millimeter at the beginning of the tip of the blades which is named as D0(diameter 1 mm) o Extending up the blades to the most coronal part of the cutting edge is termed as D16 (diameter 2-16 mm in length). o Added revisions are under way to cover instruments constructed with new materials, designs, and tapers greater than 0.02 mm/mm.

K C E T 2 0 1 1

DENTAL QUEST Your Quest For The success In Post Graduate Dental Entrance !

DENTAL

QUEST

KCET -2011 128

PEDODONTICS
189) Rotation correction of tooth can be a hazardous process if correction is required over
more than degrees.? a) 20 b) 45 c) 60 d) 90 ANS (b) 45 Degree Guys we have searched texts from Grabers to Moyers. Though we found some article where its given that rotation of more than 30 degree are quite difficult whereas at one place we found incisors can be rotated till 450 while canine can be rotated up to 300 and molars can be up to 150. we suggest you therefore to go with the Kcet key till we come out with exact reference.

K C E T 2 0 1 1

190) a) b) c)

The first sign of tooth development in human being is around? 12th week of intrauterine life 6th week of intrauterine life 6th week of life d) At birth Ans (b) 6th week of intrauterine life ( McDonald Dentistry for the child and adolescent 8th ed page no 52) Evidence of development of the human tooth can be observed as early as the sixth week of embryonic life. The first macroscopic indication of morphologic development occurs at approximately 11 weeks in utero.

191) A Crying two year old child brought to your Dental clinic would be classified under
Wright's classification of child behavior as a? a) Potentially cooperative child b) Uncooperative child c) Child lacking cooperative ability d) Timid child Ans (c) Child lacking cooperative ability ( Textbook Of Pedodontics by Shobha Tandon 2nd ed page no 143-145) Wright 1975 A.COOPERATIVE (POSITIVE BEHAVIOUR) A) Cooperative behavior b) Lacking cooperative ability :- seen in young child (0-3 years ),disabled child,physically and mentally handicapped. c) Potentially cooperative B.UNCOOPERATIVE BEHAVIOUR (NEGATIVE BEHAVIOUR) a) Uncontrolled/hysterical/incorrigible:- seen in preschool children on first dental visit b) Defiant/obstinate behavior:- seen in any age group and usually in spoilt or stuborn children These children can be made cooperative c)Tense cooperative:- borderline between positive and negative behavior d) Timid behavior/shy:- seen in over protective child at first dental visit e) Whinning type :- complaining type of behavior f)Stoic behavior:-seen in physically abused children Cooperative and accepts procedures without any facial expression.

DENTAL QUEST Your Quest For The success In Post Graduate Dental Entrance !

DENTAL

QUEST

KCET -2011 129

192) All the following materials can be used for obturating root canals of primary teeth EXCEPT a) Zinc oxide eugenol b) Gutta-percha c) Zinc oxide iodoform paste d) Calcium hydroxide and idoform paste Ans (b) Gutta-percha ( Textbook Of Pedodontics by Shobha Tandon 2nd ed page no 417) Gutta-percha is not a obturating material in primary teeth as it cannot be resorbed while the permanent tooth is erupting. 193) The first book on child dentistry was published by a) G W Wright b) Joseph Hurlock c) Gerauldy d) Robert Bunan Ans (b) Joseph hurlock ( Did not get Reff from text but guys. Found on net that Robert Bunan published the first book of child dentistry )

K C E T 2 0 1 1

194) The art of re-enforcing good behavior displayed by the child with verbal praise applies
the? a) Operant conditioning theory b) Classical conditioning theory c) Psychoanalytical theory d) Social learning theory Ans (a) Operant conditioning theory (Textbook Of Pedodontics by Shobha Tandon 2nd ed page no 127-128) OPERANT CONDITIONING BY SKINNER IN 1938 o Individual response is changed by reinforcement or extinction of previous responses. o According to this theory consequence of behavior itself acts as a stimulus and affects future behavior. o He described 4 types of operant conditioning w.r.t. the type of consequences Positive reinforcement: - occurs if a pleasant result follows the response e.g. child rewarded for good behavior following dental treatment with a gift, pat on shoulder or verbal praise. Negative reinforcement :-removal of unpleasant stimulus following a response Omission:-removal of pleasant response after a particular response.

Punishment:-introduction of an aversive stimulus into a situation to decrease the undesirable behavior. The first two reinforcements are most suitable for dental operatory

195) A mother who does NOT encourage her child to cope up with new situations thereby
making him shy and timid is an example of an? a) Overprotective mother b) Under affectionate mother c) Authoritarian mother d) Rejecting mother Ans (a) Overprotective mother

DENTAL QUEST Your Quest For The success In Post Graduate Dental Entrance !

DENTAL

QUEST
nd

KCET -2011 130

( Textbook Of Pedodontics by Shobha Tandon 2 Finn 4th ed page no 25) Mothers behavior Overprotective, dominant Over indulgent Under affectionate Rejecting authoritarian

ed page no 147 and Clinical Pedodontics:

Child behavior Shy,submissive,anxious Aggressive,demanding,display of temper tanturums Well behaved,may be unable to cooperate,shy, ,may cry easily Aggressive,overactive,disobedient,evasive and dawdling

K C E T 2 0 1 1

Exaggeration of overprotection is harmful to normal psychological development of the child. Causes of overprotection : Child conceived after a long time Only child Sick/handicapped child. Features : Child is not permitted to use his own initiative or make his decisions himself Mother takes active role in childs social activities Child is submissive,shy,timid,fears new situations and lacks self-confidence.

196)

A frightened child is exposed to a dental clinic setting which is pleasant, colorful and different from a previous clinic where a painful experience occurred. The child is happy and treatment is completed. The psychological conditioning principle used here is a) Acquisition b) Generalization c) Extinction d) Discrimination Ans (d) Discrimination ( Textbook Of Pedodontics by Shobha Tandon 2nd ed page no 126) PRINCIPLES INVOLVED IN CONDITIONING PROCESS o ACQUISITION: - learning a new response from environment by conditioning o GENERALIZATION: - here a test stimulus similar to training stimulus results in a response. E.g. a child with previous bad experience with a doctor in white coat always will associate a doctor in white coat with pain. o EXTINCTION: - occurs if association between the conditioned and the unconditioned response is not reinforced.e.g. In above example subsequent visits to a doctor without an unpleasant experience results in extinction of fear. o DISCRIMINATION:- it is opposite of generalization. Here if the child is exposed to clinic setting which is different from those associated with painful experiences than the child learns to discriminate between the two clinics and even the generalized response to any office will be extinguished. 197) The groove separating the gumpad from the palate is called a) Gingival groove b) Dental groove c) Lateral sulcus d) Transverse groove Ans (a) Gingival groove ( Textbook Of Pedodontics by Shobha Tandon 2nd ed page no 57)

DENTAL QUEST Your Quest For The success In Post Graduate Dental Entrance !

DENTAL

QUEST

KCET -2011 131

The alveolar arches of an infant are called gum pads Each gumpad is divided into ten segments by TRANSVERSE GROOVES. The grooves between the deciduous canine and first molar are prominent and called LATERAL SULCI. UPPER GUM PAD :- horseshoe shaped and shows Gingival groove :- separates gumpad from palate Dental groove :- originates in incisive papilla region and extends backwards to touch the gingival groove in the canine region and then laterally to end in the molar region Lateral sulcus :- separates canine and deciduous first molar LOWER GUM PAD :- U shaped and shows Gingival groove :- demarcates lingual extent of gum pads. Dental groove :-runs from mandible backwards and joins gingival groove in canine region Lateral sulcus :- same as above

K C E T 2 0 1 1

198) Which of the following is TRUE regarding voice control during dental treatment of children? a) It should be a monotonous soothing conversation b) It is a sudden and firm command c) It is intended to frighten the child d) It can be used on a two year old child Ans (b) It is a sudden and firm command (MC DONALDS text book Pedodontics 8TH EDITION page no 43-44) Voice Control There is a modification of pitch and own voice, hence not monotonous Should be sudden and firm It used to gain the attention and not frighten the child Used in age group of 3-6 years

MISCELLANEOUS
199) Herpes simplex virus binds to epithelium using a) Chondroitin sulphate b) Heparan sulphate c) Hyaluronidase d) Keratin sulphate Ans (b)Heparan sulphate (Lehningars biochemistry 4th edition,pg no 258,263) Herpes Simples 1 and 2 both have Lectins on the surface (the causative agents of oral and genital herpes) which bind specifically to Heparin sulfate on the cell surface as a first step in their infection cycle. 200) No surgical procedure should be performed in patients with thrombocytopenia unless the platelet count is a) >60,000 b) >70,000 c) >80,000 d) >50,000 ANS IS ABCD by RGUHS University (Textbook of Oral Surgery by Neelima Malik, 1st edition pg no 676) Normal platelet count 1, 50,000-4,50,000 per microliter of blood.

DENTAL QUEST Your Quest For The success In Post Graduate Dental Entrance !

DENTAL

QUEST

KCET -2011 132

50,000-1, 00,000 mild increase in bleeding time 20,000-50,000 easy bruising <20,000 spontaneous bleeding. Minor oral surgical procedure can be safely done above 80,000-1,00,000 count.

K C E T 2 0 1 1

DENTAL QUEST Your Quest For The success In Post Graduate Dental Entrance !

You might also like